Transcript for:
Pediatric Disorders Overview for Medical Students

Okay, good. So, welcome to the Pediatric Shelf Review. My name is Devine. I am a fourth-year medical student. I have no financial disclosures, obviously.

I'll let you read the intro on your own. So, a four-and-a-half-year-old male is brought to a well-child visit by his concerned parents. For the past four months, he has woken up agitated and in tears, usually before midnight. He goes to sleep around 9.30 p.m. He often appears drowsy.

When these episodes occur and goes right back to sleep after his mom caresses him for a few minutes, he has no memory of the event when he wakes up around 8 a.m. the next morning. The patient is in the 60th percentile for height, weight, and head circumference.

A physical exam conducted in the pediatrician's office is normal. What is the next best step in the management of this patient? So what do you think? Very good.

This is reassurance. Okay. So this kid has something known as sleep terror disorder.

Okay. Occasionally they bring out exam questions on the parasomnias. They're just sleep disorders. Okay.

In general, you don't do anything for these problems as kids tend to outgrow them. Okay. So sleep terror disorder, they will try to get you to confuse this with nightmare disorder. So in sleep terror disorder, the thing that happens is the kid has like a They wake up and they go back to sleep and then in the morning when they get up, they have no recollection of the events, okay?

And it tends to happen in stage N3 of sleep. Contrast that with nightmare disorder where the next morning the kid tends to fully recall the bad dream that they had, okay? And nightmare disorder, it occurs during REM sleep. And remember that REM sleep is associated with beta waves on an EEG, okay? But if they describe a child that sort of like moving around or walking outside the home, sort of having like sleepwalking disorder, then an exam answer may be to actually like do something like to barricade the door or something to prevent the child from harming himself or herself.

But most of these parasomnias, you just reassure the parents, the kids get better with time. Okay, next one. An 18-month-old male is brought to the ED with shortness of breath. A chest external admission is notable for a well defined consolidation.

in the left upper lung lobe. This is the child's sixth episode of pneumonia in the past nine months. He was recently placed on a course of high-dose amoxicillin and clavulanate for a severe ear infection.

His vital signs include a temperature of 102.9, heart rate of 170, and a respiratory rate of 30, so severely tachycardic and tachypneic. There is no cervical lymphadenopathy or evidence of reactive hyperplasia in the back of his throat. His mom's brother died last year from a severe strep pneumo infection.

What is the most likely cause of this patient's presentation? Exactly, very good, right? So it's B, right? So this patient has something known as Bruton's A-gamma globulinemia, right?

So it's usually from Bruton's tyrosine kinase gene mutation. So those kids tend to get recurrent bacterial infections, okay? And the pathophysiology is, again, their B cells are not proliferating. And we know that B cells are CD19 and 20 positive.

And in general, the diagnostic testing you do for this, I mean, you can do a screening with the CBC. And you will find reduced levels of every immunoglobulin. Okay.

And in general, the way you treat this is to go ahead and give IVIG. Okay. You give IVIG to replace the immunoglobulins that are missing.

So, again, kid has protons. okay, recurrent cyanopulmonary infections. It's an X-linked disorder, okay.

So in general, on exams, it should not be boys. It should not be girls that have this problem, okay. It should all be boys, okay. And you treat with IVIG injections.

And the thing is, usually these kids, they will present at more than six months of age because the IgG they get from mom in utero, okay, persists for that period of time, okay. But after six months, they begin to get into trouble. And if you notice, I mentioned strep pneumo in the question to sort of lead you towards thinking more of a B-cell problem.

OK, because the thing is, on exams, they usually give you like some bugs that the person has had infection with. And I will encourage you to use that bug pattern to decipher what kind of deficiency they potentially have. Right. So if, for example, a person has bacterial infections, primarily you're thinking of a B-cell problem.

But if you say like viral or fungal infections, you're probably thinking more about a. a T-cell problem. And if you're combining, if they give you a question about a person that has like hypocalcemia, okay, and they have like truncus arteriosus, and they have like these facial defects, you probably want to think more about the George syndrome, okay? It's a T-cell deficiency disorder, so they'll have like recurrent viral and...

fungal infections like they could have PCP pneumonia that could be an exam presentation so as a quick summary of the immunodeficiency syndromes we have Bruton's agammaglobulinemia I just talked about that x-linked BTK mutation right so it shows up just in boys they will have a lot of respiratory and GI infections okay and they will have decreased levels of every immunoglobulin and one classic exam clue may be that they have hypoplasia of lymphoid tissue right so They may say that, oh, you look in the back of the throat, you do an ENT exam, and you don't see any tonsils. Okay, that can be another classic presentation of that. Okay, and the treatment is IVIG.

IgA deficiency, it's usually like in people, kids that are much older on exams. And one presentation may be anaphylaxis with a blood transfusion. Okay, because those people, since they have the IgA deficiency, they may make other antibodies against IgA.

So if you give them blood that contains IgA, they can get a very nasty. anaphylactic reaction. And then with Scott-Aldrich syndrome, the big things you're looking for are low platelets, right? So thrombocytopenia.

They won't tell you thrombocytopenia on the exam. They'll just give you a CBC and show you that the platelet count is less than 150,000, for example, okay? And it's eczema, and it's X-linked, right?

So it has to be a boy disease on tests. And then ataxia telangiectasia, they literally have ataxia and telangiectasia, okay? The ataxia shows up first, right? So the easy way to remember that is...

A comes before T in the alphabet, okay? And they have an increased risk of lymphoma. And then SCID, severe combined immunodeficiency, right?

So it's a combined immunodeficiency. So they have B-cell problems, right? So they get recurring bacterial infections, but they also have T-cell problems, right?

So they have recurrent viral and fungal infections. And there are multiple causes of SCID, but the big ones that tend to show up on exams, I'll say probably the most important one is adenosine deaminase deficiency. An IL-2 receptor defect can also cause that. Remember, the IL-2 receptor, you'll find it on essentially every T-cell, and interleukin-2 is a T-cell stimulating factor, okay?

And again, these kids tend to get nasty, nasty infections, and it shows up very early in life, within the first few weeks, as against past six months for Bruton's A-gamma globulinemia, okay? Now, for leukocyte adhesion deficiency, the big thing you're looking for is a kid that has all these diseases where they form abscesses, but there's no pus. or they tell you that the kid has delayed separation of the umbilical cord.

If you see that you're thinking about leukocyte adhesion deficiency and it's an integrin defect like CD18. And then if you see a Bruton-like presentation but in an older kid, so like a teenager, and they have low levels of most immunoglobulins, think about CVED, combined variable immunodeficiency. It presents very similarly to this but the giveaway on your exams will be the age range.

Teenager. CVED, okay, much younger kid, like less than a year old, but greater than six months old, think about Bruton's, okay? Now, kids with SCID, right, so they present with like failure to thrive, opportunistic infections, diarrhea.

Diarrhea almost always shows up as a presenting symptom on exams, okay? And then for the George syndrome, right, so I said it's a T-cell defect, and the pathophysiology is where your third and fourth pharyngeal pouches do not develop well, right? So... You don't have parathyroid glands, right?

So you have hypocalcemia, can present that seizures. And because they also have a poor differentiation of the thymus, you won't find a thymic shadow on a chest x-ray. So no thymic shadow on a chest x-ray of a newborn could clue you into either DeGeorge syndrome, okay, or SCID. And for these patients that have these cell-mediated immune deficiencies, you want to avoid a live attenuated vaccine, okay, because those can reactivate and cause a lot of problems. Now.

Now, CGD, right? So if a person has infections with recurrent infections with catalase-positive organisms, okay, you're thinking about NADPH oxidase deficiency, right? So CGD, chronic granulomatous disease.

And remember your testing, right? So classically on exams, like in ages past, they always put the NBT test, nitro-blue tetrazoleum. But that's beginning to be de-emphasized on exams.

And they are beginning to introduce this newer test because it's pretty sensitive. the dihydrorhodamine test, okay? So that's something that you may see on an exam.

You may not see your classic NBT, okay? And again, the reason they get recurrent infections with catalase-positive organisms is that they don't have any DPH oxidase, so they cannot make superoxide radicals that can be converted to bleach that would kill off bugs, okay? So they co-opt the hydrogen peroxide that comes from bugs to make that happen.

But if you're a bug that possesses catalase which breaks down hydrogen peroxide to water and oxygen then you can thrive under that kind of environment and cause recurrent infections okay and then if you see a kid with like neurological problems and albinism albinism is the big thing you're looking for and recurring respiratory infections think of chidiak higashi disease any questions on that good Okay. Now, an 18-month-old infant is brought to a pediatrician by his mom after she noticed a bulge in his abdomen when lifting him from a crib. The presence of a left infraumbilical mass is confirmed on a physical exam.

Vital signs are notable for a blood pressure of 138 over 97. All other vitals and a spot urinalysis are within normal limits. The infant has no relevant past medical history and had a congenital hypospadia. corrected six weeks after delivery.

An abdominal CT scan is notable for unilateral, non-calcified mass. The most likely pathophysiology of this child's presentation is what? Exactly right.

So this is a Wilms tumor. OK, so Wilms tumor. Right. It's a renal derived mass. Right.

So the answer is B. OK. The thing that will try to make you confuse this with is a neuroblastoma. OK.

There are certain things that help you differentiate between Wilms tumor and a neuroblastoma on exams. OK. So Wilms tumor is usually unilateral and usually does not cross the midline. OK. But neuroblastoma, it could be unilateral.

But many times on exams, it crosses the midline. Okay. And for a Wilms tumor, it's usually not calcified on CT imaging.

Okay. But for neuroblastoma, it's usually calcified. If you see a calcified abdominal mass in a kid, think about the neuroblastoma.

So the kid has Wilms tumor, right? And remember, it has certain associations, right? Like there's the WAGR complex. You probably remember from step one where they have like a Wilms tumor. They have no iris.

They have GU anomalies and mental retardation. Okay, and the mutation is from a WT1 gene mutation on chromosome 11. Okay, and your classic triad that you learn for step one is like hypertension, hematuria, and a flank mass. Yeah, it's a classic triad, but it probably shows up in less than 10 to 30% of patients that present with a Wilms tumor.

So it's not exactly classic, but that's usually what they put on exams. And in general, you perform an ultrasound first to make the diagnosis. And then you follow that with a CT scan. And again, it's a non-calcified mass that does not cross the midline on abdominal imaging. Okay?

Neuroblastomas, they come from neurocrest cells. And they usually, they can, not always, they can produce catecholamines. And again, they're calcified and cross the midline. And... Because a neuroblastoma is a neurotumor, it can show up in the mediastinum.

In general, for exams, you want to be able to differentiate between an anterior versus a posterior mediastinal tumor. Neuroblastomas do not show up in the anterior mediastinum on exams. They show up in the posterior mediastinum because they are neurotumors.

The anterior mediastinal tumors are more like your T's, like a thymoma or a teratoma. Okay? Although most teratomas show up, most of them are sacrocoxigil.

And yeah, I think that's all I'll say with that. And Wilms tumor, right? So I said there's a WAGR syndrome association.

There's also a Beckwith-Witherman syndrome association, right? Where like one side of their body is bigger than the other, okay? They tend to get all these GI tumors like a Wilms tumor or hepatoblastoma, right?

So like a liver mass. And... they may have hyperplasia of the pancreatic islet, so they may overproduce insulin that can actually present as a hypoglycemic seizure in a neonate, right?

Because if you're secreting a ton of insulin, your blood glucose levels go down a lot. Okay, so let's do some info cluster questions, right? So let's assume we have a kid. The head circumference is less than the second percentile, okay? And you detect the symmetric intrauterine growth restriction on exam.

And the kid has a thin upper lip and a smooth philtrum. And this kid has behavioral problems. What do you think this is? Fetal alcohol syndrome. Very good.

Fetal alcohol syndrome, right? So they almost always put the smooth philtrum on exams. Okay. So this is something that's entirely preventable.

Usually see the smooth philtrum, the thin upper lip. The kid usually has a small head like a microcephaly. Okay.

And they tend to have behavioral problems like ADHD, for example. ADHD has a very strong association, excuse me, with fetal alcohol syndrome. And a few other quick things that I just placed here.

I just put like a lot of the high yield findings for Down syndrome, right? So if you're taking a pediatrics shelf, right, you should almost expect to get one or two questions on Down syndrome. They love to throw those on there. Okay, and Down syndrome, remember it has a lot of high-yield associations, right? Like the epicanthal folds, the Hirschsprung's disease where they fail to pass meconium, right?

At more than 48 hours after delivery. Duodenal atresia, right? So your classic double bubble on abdominal imaging, okay?

Or they have like a primum ASD, right? An endocardial cushion defect. And they also have an increased risk of ALL, okay?

Remember that mnemonic from step one where they say we... we all fall down okay the all with down syndrome okay and for trisomy 18 right edward syndrome uh they'll usually describe a kid with rocker bottom feet okay and they say like they have like overlapping digits and overlapping toes okay and then trisomy 13 uh pato syndrome it's more like cleft lip and palate any questions on this Okay. And this is more for an OB-GYN shelf, but increased nucleotranslucency on a prenatal ultrasound is pretty classic for Down syndrome. Okay.

And if you do a quad screen of the mom, you'll find low levels of AFP and S-trial, but the beta HCG and inhibin levels will be up. Okay. Contrast that with Edwards syndrome, where the AFP, the S-trial, and the beta HCG are all low. Okay.

And for Down syndrome, again, the genetic pathophysiology is able to recognize the words maternal non-disjunction, okay, or some kind of robotsonian translocation. I talk about the mechanisms behind that in a different podcast. Now, some other high-yield associations. Fetal alcohol syndrome also has some association with certain heart defects. Probably the most common one is a VSD, okay.

And again, they also have associations with the psychiatric problem ADHD, okay. and I differentiate between a symmetric and an asymmetric intrauterine growth restriction here. I'll let you go over that on your own.

You probably want to know the cause of a symmetric versus an asymmetric IUGR, okay? They may occasionally describe a kid that has small head, small body, small everything. You know it's a symmetric problem, okay? And then they say which of the following could be a potential cause, okay?

So you want to be able to associate that with something that's like an insult much earlier in development, okay? Like exposure. to a torch infection or fetal alcohol syndrome.

Okay. Now, a two-year-old female is brought to the ED by her mom. Physical exam is notable for bilateral, non-exudative conjunctival injection, so like bilateral red eye. HENT exam reveals cracked fissured lips, tongue erythema, and left anterior cervical lymphadenopathy. There is no lymphadenopathy on the right.

Okay, and vital signs are notable for temperature of 105. So what's the next best step in the management of this patient? IVIG and aspirin, right? So what does this kid have?

Very good. This kid has Kawasaki's disease, okay? So there's a mnemonic where they talk about like crash and burn symptoms, right?

So your C is your conjunctivitis, which this kid has, okay? The kid usually has a rash on the palms and soles, okay? Remember, Kawasaki's disease is one of the causes of a rash on the palms and soles.

And they tend to have unilateral anterior cervical lymphadenopathy, okay? They may have a strawberry tongue, okay? And they usually have very high fevers, okay?

Very high fevers. And in general, you want to treat with aspirin. Do you usually give aspirin to kids?

No, right? You don't give aspirin to kids because you're trying to avoid what syndrome? Exactly. You're trying to avoid Rice syndrome, okay? But this is one of those high-yield aspirin indications for kids, okay?

You give that to prevent the coronary artery aneurysms that are associated with Kawasaki's disease, right? Because it's a kind of vasculitis. So, rash on the palms and soles, conjunctival injection, anterocephalic lymphadenopathy, strawberry tongue, that's Kawasaki's disease.

And there are certain high-yield complications that could arise with Kawasaki's disease. One complication is having gallbladder high drops. So if they give you a question that involves a kid that has this presentation, and then they present with right upper quadrant pain, you really want to think about gallbladder high drops.

It's one of those high-yield complications that is associated with Kawasaki's disease. And again, you trade with IVIG and aspirin. You make your diagnosis or you try to survey. for coronary artery disease with an echocardiogram. You just want to look for aneurysms that may occur in the coronary vessels.

And then a closely related disorder is HSP. So if they describe a kid that has a recent upper respiratory infection and then they have abdominal pain and they have joint pain because it's a systemic manifestation of IgA nephropathy and then they have palpable puberty. below the buttocks, okay, from the buttocks going down, okay, you really want to think about HSP.

And if they then describe the child suddenly having like very nasty abdominal pain that is intermittent, where they curl up in a ball, and then after a few minutes or hours, they feel better, and then they tell you that, oh, you, and usually they'll show you like this diaper with like red, nasty, goopy poop. You really want to think about intussusception. Intussusception is a potential sequel of Hinoxial Lime Paper. Okay, so a seven-year-old male is brought to the ED by his mom after having low-grade fevers for the past two weeks. Physical exam is notable for tenderness to palpation of the large joints of the upper and lower extremities.

The patient recently went on a class trip to Hartford, Connecticut. Boom, boom, boom. In addition to confirmatory lab testing, what is the next best step in the management of this patient? It's amoxicillin, right?

So what does this kid have? Exactly, this kid has Lyme disease, okay? And in general, for Lyme disease, you treat with a tetracycline, okay?

But if a kid is less than eight years old, that's your cutoff on exams, a kid that is less than eight should not be treated for Lyme disease with a tetracycline because it can cause like tooth discoloration, okay? And it can also cause problems with the bones in kids. So in kids that are less than eight years old indicated. treatment for Lyme disease is amoxicillin, okay? Let's see, any other big thing I want to talk about here?

The only other thing I'll add is if a kid has like cardiac issues from Lyme or like neurological problems with Lyme, you probably want to go ahead and give them the third generation cephalosporin, ceftriaxone, okay? Your tetracyclines and amoxicillin won't cut it under those circumstances, okay? Although remember, if a kid is super young, right? So like one month old, like a neonate basically.

You want to avoid ceftriaxone, okay, because ceftriaxone can cause hepatic cholestesis, so that's kind of bad. So in general, for very young kids, if you want to give them a third generation cephalosporin, you proceed with cefotaxim. Cefotaxim is the very common third gen cephalosporin given to kids, okay?

And some other bad things for young kids, right, chloramphenicol can cause gray baby syndrome because kids usually have a deficiency of UDPGT. so they can metabolize chloramphenicol very well. And then Cipro, you also want to avoid it because it can cause a tendon rupture, right, like the Achilles tendon.

Okay, now a two-month-old with his head twisted to the left and his chin twisted to the right, a physical exam is notable for a mass palpated along the sternocleidomastoid muscle. What's your diagnosis? Exactly. This kid has congenital torticollis. And it's usually a pretty classic presentation where they say, oh, the kid, like they can even give you a picture where the kid's head is somewhat like this.

Okay. You want to think about congenital torticollis with that. And really the pathophysiology is in the, let's assume you have a very tenuous labor delivery process.

And then the kid sustains injury to the sternocleidomastoid muscle. It could heal with scar. And we know that scar It's not very flexible, okay? So the kid's head may be contorted in one position.

In general, the thing you try to do is to perform some kind of passive neck stretching, and that usually makes the problem go away over the passing of time. You don't want to just let it go because it could become permanent, and you may need to do like surgery and stuff like that, okay? But the first step in management is generally to just perform like neck exercises to sort of straighten out the kid's head. Any questions on this?

Okay. Now, two-year-old female heading to daycare with a babysitter is throwing tantrums by throwing herself on the ground. She's trying to pull away from the babysitter and then you hear a pop sound, right?

So let's assume the babysitter is lifting her up with this hand and hear a pop sound, okay? And the two-year-old begins to complain of pain at the elbow and the forearm is held in a prone position. What is this?

It's a nursemaid elbow. Very good. It's a nursemaid elbow.

Okay. Basically, the thing that happens is as the kid, you know the way kids play where like a person that's really tall is holding them like this. Okay.

And then as the kids are trying to pull away. radio head displaces, and then usually the kid holds their arms like this, and they say, oh, it hurts, okay? It's a pretty easy diagnosis on exams.

It's nurse-made elbow, and usually the way you treat it, you don't even need to do any kind of imaging. It's a clinical diagnosis. The story is usually pretty consistent. So all you need to do is just hypersupinate or hyperpronate the extremity, and you just pop the radio head back into place, and the problem goes away, okay?

Pretty easy question on exams, but pretty common. Now, for this question, I just want names of bugs, okay? So what's the most common overall cause of osteomyelitis? That's Staph aureus, very good.

Remember, it's a gram-positive carcass, okay? And it's catalase and coagulase positive. Okay, now, osteomyelitis in a 16-day-old neonate. Gruby strep, right? So strep agalactiae, it tends to cause most...

notable infections within the first 28 days of life like meningitis or sepsis or pneumonia. Okay. Now, a two-year-old male with left ankle osteomyelitis, and let's assume he recently stepped on a sharp substance.

Pseudomonas, right? Because your canvas or your shoes, right? They're all teeming with pseudomonas.

So if you get this exam story where a person steps on a nail and then you have like a bone infection afterwards, think about pseudomonas, okay? Now, let's assume we have a kid with right knee osteomyelitis and you perform abdominal imaging or you do like a physical exam and you cannot palpate the spleen. Let's assume this kid has auto-infarcted his spleen.

What would that be? That would be Salmonella. So what does this kid have? Sickle cell disease. Exactly, right?

So if you get a sickle cell patient with osteomyelitis, you really want to think about Salmonella. Okay, so in general, if an exam question tells you that, oh, like you find like a red joint or whatever, your first step in management always, always, always is to tap that joint, okay? Get an arthrocentesis.

And if you're really suspected in osteo, you can get like an x-ray and say, okay, let me see if I can find like signs of infection and inflammation. But if the x-ray is negative, you probably want to jump to an MRI. An MRI is pretty sensitive for osteomyelitis.

And you usually want to get some kind of bone culture to detect the bug so that that can direct your antibiotic treatment. And for osteomyelitis, you need to treat for a very long time, usually for weeks on end to completely eradicate the infection so that the kid does not have any kind of bone loss. Okay, now, seven-year-old male with fevers for the past six weeks. He has two febrile episodes per day, okay? And this, you perform a physical exam, you know, diffuse cervical lymphadenopathy and hepatosplenomegaly.

And the mom says that he has a salmon-colored rash that seems to come with the fevers, and then it remits when the fevers go away. So what is this? This is Stills disease, right? So this is a systemic arthritis, okay? It's one of the kinds of juvenile idiopathic arthritis, okay?

You definitely want to know your rheumatoid diseases in kids for your exam, okay? The classic presentation on exams is like hepatosplenomegaly, recurring fevers, and they tell you that the kid, as the fevers come, an evanescent, right? That's the buzzword. An evanescent rash shows up and then disappears as the fevers are going.

If you see that, you really want to think about Stills disease. And usually your treatment is just an NSAID like ibuprofen, okay? That's usually good for most of them. But if it does not resolve, then you want to go on to the bigger stuff like methotrexate. Methotrexate, remember from step one, it's a dihydrofoli reductase inhibitor, right?

So as an addendum question, I guess, if a kid gets methotrexate, right, and they begin to have severe bone marrow suppression, on your exam, they want you to pick lucovorin as the rescue agent with that. to sort of bring the bone marrow back, okay, because methotrexate is a folate antagonist. Now, a few quick words on the other kinds of GIA, right?

So the oligarticular GIA, okay, it affects only one to four joints, okay? And they're just, I'll say there are certain like high yield associations you want to know with each kind of GIA. So for the oligarticular kind, the one you really want to pay attention to is anterior uveitis.

So these kids deserve regular slit lamp exams, okay? So you can catch it early so they don't lose their vision. And this one usually has a very strong association with ANA positivity.

Now, if you have more than five or more joints affected, that's polyarticular, GIA, okay? And this one is pretty destructive. And it's the one that has an association with RF positivity or RF negativity, okay? It has... the RF positive sub-5, it's really, really nasty, destroys a lot of joints, okay?

And it has a pretty strong semblance actually to adult rheumatoid arthritis, okay? And if a kid has dactylitis, right? So like swollen digits and it hurts, one thing you want to think about is sickle cell disease. That's probably one of the more common causes of that, but don't discount psoriatic arthritis in a kid, right?

So a common MBME strategy is to give you... old people diseases in young kids, okay? So if it looks pretty classic, like if they show you like the psoriatic form lesions, right, like the silver scale on the extensor surfaces, and they also tell you that this kid has arthritis, okay, and they show you like, let's say like a hand x-ray that shows the pencil in cup deformity, you really want to think about psoriatic arthritis. Don't be afraid to pick that answer on exams.

Now... So these are two different questions, and I want you to give me the diagnosis, right? So the first one is a seven-year-old male recently had a URI.

He presents with a painful limp that began yesterday, right? So recent URI and then painful limp, okay? Why count as normal? ESR is marginally elevated. There is no evidence of joint erythema or effusion on physical exam.

What is this? Exactly, right? So transient, excuse me, transient synovitis, okay?

So the classic presentation here is like a recent viral upper respiratory infection, and then the kid just has a painful limp, right? Like some mild joint inflammation. Usually, if you do a physical exam, you don't see anything on the joints, okay?

So for these kids, you just basically give them NSAIDs, and they should be fine. You don't need to tap the joint. But if you see in the exam questionnaire, they say that, oh, these... Mention a specific joint that is like red and hot, you want to go ahead and do an atherocentesis. But if there are no joint findings, you don't need to tap that joint.

Now, a four-year-old female is brought to the ED by concerned parents. She's completely fine during the day, but cries to sleep at night from bilateral calf pain. Her physical exam is completely normal.

She runs around and plays with her friends during the day. Exactly, right? These are groin pains, right?

So it's a pretty classic presentation. Basically, they'll tell you about a kid that's pretty playful during the day, has no pain during the day, and then they'll talk about having bilateral calf pain at night, okay? If you see that, think about groin pains.

And if they show you a question about a kid that has not bilateral pain in this case, but unilateral pain, right? Like pain at a specific region, and they say that the pain is worse at night and relieved with NSAIDs. Okay, think about an osteoid or stoma. Okay, usually you will be able to palpate like a like an actual bone mass on a physical exam Okay, the pathophysiology behind the pain is you have a lot of release of prostaglandins So given a Cox inhibitor like an NSAID will decrease the synthesis of that prostaglandin and take away the pain Now, some other things that can cause joint problems in kids, right?

So if kids have hemophilia A and B, okay, remember these are X-linked recessive disorders, okay? Remember your step one mnemonic, right? Like hemophilia A is hemophilia 8, and hemophilia B is B9, okay?

Like B9, okay? So these problems, right, if kids begin to bleed into their joints, they can actually have an arthritis from that, okay? So hematosis, bleeding into the joint, it can trigger an arthritis. Now... if you see a kid that's less than a year old, right, with like fractures, or they say like, oh, you see fractures in different stages of healing, or spiral fractures, your next step in management on the exam is probably to like sort of hold the kid in the hospital and call child protective services, okay?

That's pretty concerning for child abuse. And then if you have like a kid, a male kid that's obese, okay, usually it's a teenager, and they tell you that, oh, the teenager has like hip pain, you want to think about skiffing, okay? slipped capital femoral epiphysis. In general, you just perform an x-ray to make the diagnosis.

It's like the ice cream falling off a cone appearance on imaging. And the way you treat this is you actually have to perform surgical pinning so that you can keep those two parts of the joint together. I'm not really going to spend time on the Salter-Harris fractures. I think those are relatively low yield. And then developmental dysplasia of the hip.

So if you get a question about a kid with a clicky hip, On exam, a newborn with a clicky hip, think about DDH, developmental hip dysplasia. And there are many maneuvers that you can use to sort of support your diagnosis, like the Barlow or the Orolani maneuver. The Barlow maneuver displaces the joint from the acetabulum. The Orolani maneuver brings the bone back into the acetabulum. If you find those positive findings, you just want to go ahead and place the kid on a public harness.

It's kind of uncomfortable, but it resolves the disorder over a couple of weeks. Now, Leukavipirthes disease, right? So if you notice, I'm sort of just running through the pediatric ortho disorders, okay?

So Leukavipirthes disease, they will describe a child that has like an insidious onset of heat pain, okay? And usually they'll say that the kid walks with an antalgic gait. If you see that, think about LCPD. It usually shows up in much younger kids compared with SCIFI. SCIFI tends to show up in kids that are like in their teenage years versus LCPD that tends to show up in kids that are like five to eight years old on exams.

And this is an avascular necrosis of the femoral head. Sometimes on exams, they'll describe the thing and you're reading the question, you're like, oh yeah, I think it's like calviparthes disease. And then they ask for the mechanism in the question. Okay.

It's an avascular necrosis that's behind this. Okay. And then if they describe a kid and you see the buzzword pain over the anterior tibial tubercle, uh, you really want to think about Osgood-Schlatter disease. Okay.

It's just from, if the kid is super active in sports, as they keep, uh, uh, pulling the, uh, uh, Pulling the patellar tendon over the tibial tubercle, you can have some kind of inflammation. That's why it's called a traction of apophysitis, okay? So pain over the anterior tibial tubercle, that's Osgood-Schlatter.

Usually those kids just take NSAIDs, rest, and usually the problem will go away, okay? And then if a kid has clubfoot, you just perform serum manipulation and castin. You don't necessarily need to do surgery. You only do surgery or proceed to more invasive things.

if these measures don't work, okay? And then for other pediatric ortho problems, remember the retinoblastoma gene mutation, okay, with osteosarcoma, okay? And then E-wings with 11-22 translocation and the onion skin in appearance on bone X-ray.

Okay, now a neonate failed to pass meconium at birth, has rectal prolapse as a teenager, along with failure to thrive. This kid is infertile and has recurrent respiratory infections. They have diabetes and elevated PTH. What is this?

Right. So this is cystic fibrosis. Right.

So cystic fibrosis. Remember, it's an autosomal recessive disease. OK. It's a CFTR gene mutation.

The most common one is the Delta F508 mutation. OK. And because these kids have their pancreatic ducts clogged off.

OK. They can develop pancreatic insufficiency and actually get diabetes. with that because the pancreas is non-functional.

And because they are not releasing pancreatic lipase, okay, those kids also do not, they begin to develop deficiencies of fat-soluble vitamins, like vitamin A, so it can present with visual problems, or vitamin D, okay, so they can become hypocalcemic and develop a secondary hyperparathyroidism with that, okay. They can have bleeding problems because they have a lack of vitamin K. Okay?

And these kids, they tend to get recurring respiratory infections. And on exams, if the kid is less than 20 years old and they have a pneumonia, you really want to think about staph aureus as the cause. Okay?

But once they are older than 20 years old, you want to begin to think more about pseudomonas. Okay? So, less than 20, staph aureus is the most common cause of respiratory infections in kids with cystic fibrosis.

But over 20, you're going more with pseudomonas. And in general, you'll treat... Give them back what they're missing, right? So you give them back pancreatic enzymes, pulmonary toilet, you know what that means.

Dornay's alpha breaks up phosphodiester bonds in DNA, okay? So that you can break up the nasty goopy stuff that sort of clogs up their airways. And then you want to give them like bronchodilators, tobramycin, right? It's an aminoglycoside, so it covers gram negatives like pseudomonas pretty well, okay?

And don't forget your sweat chloride test, okay? And these kids, they may even give you a question about a kid that has recurrent respiratory infections and nasal polyps. If you see that, you really want to think about cystic fibrosis on exams. Now, respiratory infections.

So these are my aftermath slides for respiratory infections. On the pediatrics shelf, there are certain things you want to pay attention to. You want to ask yourself if they talk about a wheeze or stridor on the exam.

If they talk about stridor, you're thinking of an upper respiratory infection. If they describe wheeze, you're thinking more about a lower respiratory tract infection. And if they describe a wheeze and it's a kid that's less than two years old, you're thinking about RSV, the respiratory syncytial virus. And usually they will describe the kid that's having this wheeze. And they'll say like, oh, days before, the kid had like runny nose and they had like...

like a mild fever and stuff like that. If you see that in a kid, especially a kid that's less than two years old, you really want to think about RSV. And the thing with RSV is that, in fact, I will just say this as a general principle. When you're taking the PEATS shelf, you want to make sure that you look at the age range. There are just certain things that don't show up in certain age ranges on the exam.

So if a kid is less than two years old, they probably do not have Group A strep. pharyngitis. It is pretty, pretty rare in that age range, okay? So respiratory infection, especially a lower respiratory tract infection in a young kid, less than two, think more about RSV, okay? Now, like I said, Strider should help you think more about an upper respiratory tract infection, okay?

And your two big uppers you want to think about on the exam is epiglottitis, okay? So it will be a very acute presentation, super high fevers. And they may tell you that the kid is tripodting, right? So the kid is like, oh, I'm trying to get in air that way. Another one could also be CRUP, laryngotracheobronchitis, right?

This is classically caused by the parainfluenza. virus, okay? And for RSV bronchiolitis, you just supportive care, give them humidified oxygen. Some people used to give ribavirin, but it's pretty controversial.

I'll say I've probably never seen anyone give ribavirin to a kid with RSV, okay? But if a kid is high-risk, let's assume they have like cystic fibrosis, right? And you want to prophylax, if you may, against RSV in those populations, right? So kids that have some kind of immunodeficiency disease, that increases the risk of recurring respiratory infections, you'll probably want to give them the monoclonal antibody against RSV, okay, like palavizumab.

And remember, do not pick bronchodilators for RSV. It does not work, okay? It does not work. There are many studies that have shown that bronchodilators do not help in RSV.

Now, croup, right, it's an upper respiratory infection, okay? And if you're comparing croup with, um, with epiglottitis, They are both upper respiratory tract infections, but epiglottitis is a supraglottic infection. You get supraglottic inflammation. Contrast that with a croup that is more subglottic inflammation.

So those words, supraglottic, above the glottis, or subglottic, below the glottis, those are things that occasionally pop up on exams. So if a kid has inspiratory stridor and they have a viral prodrome, And then they describe that you have like a barky, seal-like cough. You really want to think about croup.

And again, it's usually caused by the parainfluenza virus. And the symptoms classically on exams are worse at night. Okay? And your treatment there is racemic epinephrine.

Okay? Epinephrine will open up their airways. But remember, bronchodilators do not help for RSV, but they do help for croup.

Now, epiglottitis. Pretty classic exam question. If you perform a chest x-ray, right, you can see the thumbprint sign.

from all the inflammation. Don't touch the child. Don't try to give antibiotics or whatever. No, just keep them comfortable.

Your next step in management on exams, 100% of the time, is to prepare for emergent intubation. And it's not common. It's not that common anymore because we now vaccinate most kids against Haemophilus influenzae B.

Okay, and if a kid though like has like no fever nothing and they say like oh like 30 minutes ago The kids suddenly had respiratory distress. That's not an infection. Okay, that's a foreign body aspiration I usually just put in a flexible scope and try to get rid of whatever is causing the airway obstruction Now another respiratory pathology that could present on exams is quenal atresia. Okay This could actually show up as part of the charge association. Remember, there are all these words with associations in peds.

There's like the charge association. There's the vectoral association. The charge association has, may have a quenal atresia, okay?

And basically, you basically have an obstruction at the back of the nose, okay? So these kids, when they're like normal and they're not crying, they're cyanotic because kids in general obligate nose breathers, okay? But when they begin to cry, they open up their mouth, And that can bring in oxygen.

So if you get a question about a kid that is blue at rest, but pink with when the kid is perturbed and crying and everything, think about coenal atresia. Now, a five-month-old is brought to the ED by his mom. The infant's parents have observed what appears to be jerky movements of the head, arms, and legs for the past two weeks.

These episodes occur multiple times a day. Over short time periods, his medical history is notable for gross and fine motor developmental delay. Physical exam reveals hypopigmented macules that are more prominent on the trunk. The rest of the physical exam is within normal limits.

What is the next best step in the management of this patient? E, right? So, what does this patient have? What kind of seizure disorder?

So they have tuberous sclerosis, which classically presents with West's syndrome. Infantile spasms is a kind of seizure disorder that's pretty common in tuberous sclerosis. And usually on exams, the agent is usually less than a year old.

And the kid has generalized seizures where they're sort of like seizing. They can have these seizures like hundreds of times a day. It's actually pretty severe. And the way you treat this is with ACTH. If you don't see ACTH on a test, proceed with vigabatrin.

Those are two very good treatment options for West syndrome. And another big thing you also want to remember. So remember, ACTH comes from pro-opio melanocortin, right?

Just like MSH, melanocyte-stimulating hormone, right? So if you're remembering back from step one, if a person has Addison's disease and the adrenal glands are not working, right? And they're over-secreting ACTH, they also get skin hyperpigmentation because they're also releasing a lot of MSH alongside.

So the answer is E. So West syndrome. You don't need to be able to recognize a hypsarrhythmia on EEG, but you should be able to know the buzzword. That's the buzzword finding on EEG in a hypsarrhythmia. And then if you have a kid with many different kinds of seizures, and they say like a 2.5-hertz slow and spike wave pattern, think about Lennox-Gastaut syndrome.

If you get more of a kid, so note the difference in age ranges. This is usually in kids less than a year old. This is in kids like two to six years old.

And then if you get a teenager, right, so a person between like 11 and like 16 on exams, and they have recurrent like generalized seizures, think about juvenile myoclonic epilepsy, okay? It actually, for the most part, tends to have a good prognosis in kids. And then for your absent seizures, right, don't forget your three-hertz spike and wave pattern, okay? And they will describe a kid with multiple staring episodes in class. Your treatment is ethosuxamide.

Remember, it's a T-type calcium channel blocker. Now, febrile seizures, right? So they will describe a kid with like some kind of infection, very high fever, and they have a generalized seizure, okay? You don't need to do anything heroic for a febrile seizure, okay?

Just give them an NSAID, and they should be fine. Although they have a pretty high risk of recurrence of the seizure. But it doesn't necessarily mean that they'll get some kind of epileptiform. like a chronic epileptiform disorder from that. Now, if you have a typical febrile seizure, you generally don't do any imaging.

You don't need to administer any anti-epileptic drug. But here's the kicker. If a kid has an atypical febrile seizure, and there are certain criteria for that, like if the kid has a focal seizure, right?

So a typical febrile seizure is usually a generalized seizure. But if the kid has like a seizure just on one part of the body, okay, that is an atypical febrile seizure. For an atypical febrile seizure, you probably want to do some kind of neural workup, okay?

Or if the kid has a seizure that lasts more than 15 minutes, or they have neurological deficits, or they have more than one in a day, right? So remember your one-in-one rule. More than one seizure in one day, okay?

Those are atypical febrile seizures. Usually, you want to do some kind of neural workup, okay? Because those are more higher risk than the typical kind. And then for cerebral palsy, usually those kids tend to be pretty spastic. You can try to give them muscle relaxants like baclofen.

Baclofen is a GABA-B receptor agonist, or it can give dantrolene, right? It prevents the release of calcium from the sarcoplasmic reticulum. Or if it's a pretty focal spasm, you could inject the Botox. Now, next question. Mutation in chromosome 17. Physical exam is notable for iris hematomas.

freckles under the axillary, right? So axillary freckling and multiple growths on the body. There are hyperpigmented macules, right? So cafe au lait spots on the skin.

And this kid has a history of a tumor causing an afferent pupillary defect. What is this? Very good. This is NF1, okay? It's also known as von Recklinghausen's disease, okay?

It's a chromosome 17 problem. And it so happens that von Recklinghausen has 17 letters in the name. Okay. So these kids, they tend to have like lesh nodules. Those are your hematomas in the iris.

Okay. They tend to get the hyperpigmented lesions. Those are your cafe au lait spots.

Contrast that with the hypopigmented lesions that you find in tuberous sclerosis. Although both of these diseases are inherited in an autosomal dominant fashion. Okay.

And these kids have an increased risk of multiple tumor types. Like they can have a glioma of the optic nerve and that can present as an afferent pupillary defect. So those kids need relatively regular eye exams, okay? And if you're thinking more NF2 territory, think about bilateral sensory neurohearing loss, okay?

From a cranial nerve 8 problem, okay? Like a bilateral acoustic neuroma. And these kids, again, they also have increases of like meningiomas. They could also present with feels, okay? Feel chromocytomas have a pretty strong association on neurofibromatosis.

Any questions on this? Okay. Now, let's, so these are trigger questions. So let's just talk about why these kids have hypertension.

Okay. So regular garden variety, 15 year old female who recently became sexually active. OCPs, right? So OCP, in fact, OCPs are probably one of the most common causes of hypertension in women that are like less than 35 years old.

Okay. Now, 16 year old male with an abdominal mass and episodic headache. Okay. Now, urinary metanephrines are elevated. What is this?

That's a pheochromocytoma, okay? Remember, your first step in diagnosis is to go ahead and check the urine for metanephrines. So it's a tumor that secretes catecholamines.

That's why they get like the episodic headache and episodic runs of hypertension. And remember the association with your MEN syndrome, it's like 2A and 2B, okay? And also neurofibromatosis. Now, 16-year-old female with a normal plasma-aldosterone-to-ranian ratio.

Plasma, renin, and aldosterone are elevated. Fibromuscular dysplasia, right? So the, classically on exams, they will say that the kid has an abdominal brewery, like a flank brewery detected on auscultation of the abdomen, okay? So the renin and aldosterone will be elevated because those kids, they have constriction of the afrin arteriole, right?

So they are hypoperfusing the afrin arteriole, so the GG cells realize and secrete a ton of renin. Okay, so your renin and your aldosterone levels go up. Now, 16-year-old male with an elevated plasma-aldosterone-to-renin ratio, plasma renin is extremely low.

And let me tell you that this kid also has hypokalemia. Very good. This will be Kahn's syndrome.

Okay, so it's an adrenal adenoma that secretes aldosterone. Okay, so because you're secreting aldosterone, you'll increase the activity of that innate channel at the principal cell of the collecting duct. So these kids will be hypokalemic. And because they get hypertensive, that appropriately suppresses the renin, right?

So, notice the difference in the plasma-aldosterone-to-renin ratios between fibromuscular dysplasia and Kahn syndrome. Now, 11-year-old female with elevated blood pressure in her arms, her dorsalis pedis pulses are not palpable bilaterally. Very good.

This is aortic coarctation, right? So, this is probably a kid with Turner syndrome. Turner syndrome has an association of many disorders, like aortic coarctation. Now a five-year-old female with a thoracic CT revealing a posterior mediastinal mass.

We've talked about this already. That's a neuroblastoma, right? Again, it's a neural tumor, so it tends to show up in the posterior mediastinum versus your teratomas, your thymomas, and lymphomas, excuse me, that tend to show up more in the anterior mediastinum. Now a 12-year-old female with AFib. There's an endocrine disorder that has a stronger association with AFib.

Very good, right? Some kind of hyperthyroidism. So let's assume this kid maybe has like Graves disease.

Who knows? Now, 8th month old female with ambiguous genitalia. Exactly right.

So they may have some kind of congenital adrenal hyperplasia, okay? The most common cause though of congenital adrenal hyperplasia is a 21-hydroxylase deficiency, okay? But that one does not present with hypertension. Contrast that with 11 beta-hydroxylase deficiency that presents with hypertension, okay, with hypokalemia, and also with ambiguous genitalia, because 11-deoxycorticosterone has some partial agonist activity at mineralocorticoid receptors. So those are your answers.

Let's keep going. Now, some quick renal considerations, right? So I've sort of talked about this already, right? So if a kid recently, like two to six days ago, had like an upper respiratory infection, and then they present with hematuria. and like edema, like the findings of nephritic syndrome, think about IgA nephropathy as the cause.

Remember, it's a nephritic syndrome, so you lose less than three and a half grams of protein per day in the urine. But if you see this same presentation two to six weeks after an upper respiratory infection, and they say that, oh, you have positive ASO titers or positive anti-DNAsB titers, especially from like a strep skin infection, you really want to think about post-treptococcal or post-infectious glomerulonephritis, okay? And these kids tend to have very low levels of C3 and C4 because the antigen-antibody complex you form in the disease activates complement. So that really lowers and suppresses your complement levels. Now, if they describe, if they give you an IgE and a frappetish presentation, if you may, and again, abdominal pain, palpable proper below the buttocks, okay?

Think about HSP, okay? It's the systemic vasculitis. That's associated with IgA nephropathy.

And then another kidney problem, right? So if they describe a kid with like visual problems, okay, and ear problems, okay, and the kid has renal issues, think about Alport syndrome, right? So there's a pretty classic triad there, right?

So like can't see, can't pee, and can't hear, high C, okay? So ear problems, eye problems, and kidney problems. That's Alport syndrome.

And it's a call for a 5G mutation. That's implicated. It's an X-linked dominant disorder. And then if they describe a kid that has more than three and a half grams of protein in his urine every day.

OK, and they have like the hyperlipidemia and they have the generalized edema and all that stuff that you think of with nephrotic syndrome. And they make it like a pretty. non-described question, think about minimal chain disease.

It's the most common cause of nephrotic syndrome in kids, okay? And if they describe a kid that has like minimal chain disease and they see, oh, they have sudden onset like flank pain, you really want to think about renal vein thrombosis, okay? Because kids that have nephrotic syndrome, right, they're peeing out a ton of antithrombin 3 in the urine, and antithrombin 3 is a factor 10 and 2 inhibitor, okay?

So they're hypercoagulable, if you may. So they can have thrombosis of certain, like, weird things. Classically on exams, it's a renal vein thrombosis. And HUS, right, so like your hemolytic anemia, low platelets and renal failure, E.

coli 0157H7. Don't give those kids antibiotics, right, because the more you kill off those bugs, the more toxin is released, okay, and the more injury you get to endothelial cells. I believe I explained the pathophysiology on the next slide.

But for purposes of time, I will keep going. Okay, now, all the quick things here. These are pretty innocuous questions.

They're not things you really think about, but they're things that may show up on a test. You want to be able to differentiate. Don't forget these other causes of proteinuria.

So if they describe a kid that just went for a big workout or they just came over a pretty serious illness, think about transient proteinuria. It's pretty self-limited. It resolves on its own.

It's not a pathologic problem, okay? Or if they describe a kid where at the end of the day, there's more protein in the kid's urine, but when the kid wakes up in the morning, you check the urine and you don't see as much protein, think about orthostatic proteinuria. Again, it's a benign condition.

It's pretty self-limited, okay? Now, let's go over a few more questions here. So two-day-old is brought to the pediatric surgery suite prior to a TEF repair.

So at... T. Festula repair. He was born at 30 weeks gestation.

Five minutes after entering into the suite, the neonate skin appears muddled and the room temperature is pretty low. So it's pretty cold. What is this? Cutis mamurada, right? So it's just from very little kids.

They have, especially preemies, they have a lot of problems with temperature regulation. Okay. It's a pretty benign. disorder. You don't need to do anything.

Probably just warm the baby and put them like in swaddles or something. Now, five-day-old African-American new need, so not the race, is brought for his one-week postnatal appointment. Physical exam is notable for gray macules over the intragluteal cleft, right? So just that region just above the buttocks.

The med student alerts child protective services. Rest of the physical exam is normal. What is this?

So it's a Mongolian spot. You don't need to do anything with this. It's a pretty common condition in African-American neonates.

It's not something that you need to call CPS for. Now, two-month-old infant is brought to the pediatrician by her concerned mom. Physical exam is notable for a raised red lesion just above her left eyelid.

What is the next best step in management? Exactly right. So this is a capillary hemangioma.

You don't need to do anything. In general, the only hemangiomas you do stop for are hemangiomas that are in dangerous spots like the airway. For those, you may have to resect so that they don't cause an airway compromise. Okay, last one.

G1P1 female is concerned about a rash she noticed on her newborn's leg and ankle. Physical exam is notable for erythematous, yellow-white papules and postules on the extremities. What is this? It's something you've probably seen on your rotation already, right?

So ETN, right? Usually pediatricians just call it ETN in the presence of patients. I mean, in the presence of parents, right? Because if they say erythema toxicum, parents are like, oh, what is that? Is that something bad, right?

So it's ETN. It's pretty benign, resolves on its own. You don't need to do anything for that, okay? And in general, for ETN, if you perform a biopsy, you actually find a lot of eosinophils. in those lesions.

It's one of those things you think like, oh, this must be low yield. Well, think again. It's pretty high yield to actually know that for exams. Now, a two-hour-old neonate born at 31 weeks gestation is transferred to the NICU for increasing oxygen requirements. A chest x-ray obtained a few minutes before transfer is notable for a long hypoinflation, so low lung volumes, with obscured left and right-sided heart borders, in addition to empiric therapy with a cholesterol derivative.

Positive pressure ventilation is started. What is the most likely etiology of this neonate's condition? This question is kind of tricky.

So this is E. So what does this kid have? RDS. Very good, right?

So this kid has respiratory distress syndrome, okay? So they have a type 2 pneumocyte problem, so they are not making surfactant. And because they're not making surfactant, right, they have increased surface tension, so their lungs collapse.

So that's the pathophysiology behind the hypoxia. So this is RDS, okay? It's a surfactant deficiency, so you have failed maturation or inadequate maturation of your type 2, not type 1, type 2 pneumocytes. So this kid becomes hypoxic, so they have an increased work of breathing.

Now, in general, the treatment is, sometimes if the kids have very bad respiratory insufficiency, you may have to intubate them, place them on a vent, and you use long protective strategies. So you use adequate PEEP, but you use very low tidal volumes. And you can actually literally deliver surfactants down the ET tube. And the associations you want to know here, the thing I will definitely say to look out for is on the exam. They may see the chest x-ray shows low lung volumes, right?

So like lung hypoinfliction. And usually it's like a wide out lung on a chest x-ray. If you see that in a preemie, think about RDS.

Okay, if a kid was not a preemie, they can't have RDS on an exam. Now, contrast that with meconium aspiration syndrome, right? So if a kid is post-term, right? So... The biggest risk factor for RDS is preterm delivery, okay?

But for MAS, meconium aspiration syndrome, it's more postterm delivery. That's the big risk factor, okay? And again, how do you differentiate this from RDS? RDS is associated with low lung volumes. MAS is associated with high lung volumes, so lung hyperinflation, okay?

And again, the age range will tell you something. Preemie, that's RDS. Postterm, that's MAS, okay?

And then another one that's... like it's usually benign, is transient tachypnea of the newborn, okay? The name is very descriptive. These kids just have tachypnea, right?

So they have a very high respiratory rate. And the problem here is that they have delayed resorption of lung fluid, right? Because they either got a C-section, because remember, there's these thoughts that as the kid is going through the birth canal, as it's being squeezed, that helps get rid of lung fluid, right? So you're going to describe a kid on exams that either had a very quick... like delivery, so not enough squeeze, or usually on exams, I will say the kids tend to be products of like a cesarean delivery.

Okay, if you see that, think about TTN. Usually, it resolves within a couple of hours. Next question, a five-week-old male is brought to the ED by his mom, who reports continuous emesis for the past four days. Blood pressure is 60 over 40, heart rate is 190, temperature is 99, okay, and the respiratory rate is 25 beats per minute.

Physical exam is notable for a sunken fontanelle and a succussion splash. Succussion splash is code word on exams for gastric outlet obstruction. Okay. So a succussion splash, an auscultation of the epigastrium. Further lab testing would most likely reveal what?

Very good, right? So what does this kid have? Pyloric stenosis, very good, right?

So pyloric stenosis where the junction between the stomach and the small intestine is basically occluded, okay? And there are certain high-yield risk factors you want to know for your test, right? So like these kids, like being a firstborn male, okay, or exposure to erythromycin in the first few weeks of life, okay, are very big risk factors for...

for pyloric stenosis. And generally, you just replenish the electrolytes, right? So they may say, what's the next step in management? And an answer may be to replenish electrolytes.

An answer may be immediate, like laparotomy or whatever. Don't pick the surgical option. You need to fix the electrolytes before you go to surgery, even though those kids may not make it out of surgery, okay? And pyloric stenosis tends to present with non-bilious vomiting, right?

Versus duodenal atresia that tends to be more associated with bilious vomiting. Because. In pyloric stenosis, the problem is proximal to the small intestine, okay?

But in duodenal atresia, the problem is usually distal to the sphincter of O.D., okay? So you can have bilious vomiting in duodenal atresia. Now, because they are vomiting a lot, right, they are losing acid, right?

So they are hypochlorimic, and they get a metabolic alkalosis. And because they are volume down, right, the activity of the renin-angiotensin-aldosterone system goes up. So they become hypokalemic.

Okay, so I've put some other things there. like the olive-like mass, you may palpate, they may say like a pulsating mass on the abdomen, but they don't usually put that on exams anymore. Okay, so my aftermath slides on GI pathologies, right?

So intussusception, again, if a kid has not constant abdominal pain, episodic abdominal pain, where the kid like curls into a ball, and they may show you a diaper with like red poop, okay, think about intussusception. Okay. In general, the way you treat intraception is you perform something known as an air enema. Okay. It's diagnostic and it's also therapeutic.

Okay. Although if they describe the kid where if they say, oh, you're seeing the current jelly stones, then that means that kid probably has like a GI tract necrosis. Okay. So for that, you may want to maybe proceed to surgery.

And usually you make the diagnosis with, you can perform an ultrasound and you'll see a target sign on ultrasound. I will encourage you to look up. a picture of this because a picture like that may show up on an exam.

Now there are certain risk factors for intussusception like a Meckel's diverticulum is one big risk factor or a history of like some GI infection. In general if a kid has a history of intussusception you want to avoid the rotavirus vaccine. If a kid has a history of Meckel's diverticulum you also probably want to avoid the rotavirus vaccine.

So one of those just weird things that may show up on exams. And then here I talk about malrotation and volvulus. I will let you read that on your own. For those, you just made the diagnosis with an upper GI series, and you need surgery to fix that.

Okay, now, a 40-year-old newborn has not pooped since birth. That's not good. You're supposed to poop before 48 hours. Now, physical exam is notable for abdominal distension, and an in-utero ultrasound was notable for endocardial cushion defects.

So this kid has Hirschsprung. So this kid most likely has what syndrome? Down syndrome.

Very good. So this kid has Hirschsprung's disease. So neurocrest cells did not migrate to the distal GI tract. So they have basically like a functional GI obstruction, if you may.

So this kid has Hirschsprung's disease. Okay. I've talked about the pathophysiology.

You don't have a Meissner or Albach's plexus. So these are one of those basic science-y things that they may put on the more clinical third-year exams. Okay.

And again, Down syndrome, remember your high-yield associations, right? So like Hirschpronk's, double bubble on abdominal imaging. That's duodenal atresia.

Remember, that's usually bilious vomiting. They may have celiac disease. Remember, they have an increased risk of early Alzheimer's because they have three copies. of chromosome 21, which contains the pre-seniline gene, okay? And before these kids start sports, okay, you need to perform a lateral neck x-ray to look for something known as atlantoaxial instability, so that they don't have like spinal cord compression, okay?

And I'll just go ahead and see this as well. Because this is probably a bonus, I guess, for your medicine shelf or whatever, Step 2CK in the future. If a patient has a long history of rheumatoid arthritis, okay, before those patients go for surgery, you also want to perform a lateral neck x-ray, okay?

Those people actually have a pretty high risk of atlantoaxial instability as well. Okay, because in the process of moving the patient around for surgery, you can trigger like cord compression and it could be fatal. Okay, so in general, Down syndrome, long history of rheumatoid arthritis, those people deserve a lateral neck x-ray to detect that lanto-axial instability.

Okay, now recent history of a febrile adenovirus infection. Home aspirin, that's not good. Home aspirin was used to treat the fevers.

The infant presents today with copious vomiting, and he ultimately becomes comatose, and the blood glucose is really low. What is this? Rice syndrome, right? So again, never give aspirin to kids.

The only time you should give aspirin to a kid on exams is a kid with what vasculitis? Kawasaki is very good. Okay, so this is Rice Syndrome.

The classic presentation is neurological problems and liver problems, right? So hypoglycemia with usually a comatose kid on exams, that's Rice Syndrome. Okay, again, don't give aspirin to kids. Now, Neone, born at 30 weeks, presents with bilious vomiting and diarrhea.

She had her first course of breast milk five days ago. Okay, so bilious vomiting. Physical exam is notable for abdominal distension and notice this is a preemie.

Thank you. Severely hypotensive. Blood pH is 7.1, so they're becoming acidemic, and they're hyponatremic. Okay, and abdominal x-rays show free air under the diaphragm. What is this?

Very good, right? So next, so if they describe a kid, and they show you, they may show you like an abdominal x-ray, and you see the classic finding pneumatosis intestinalis, where you find air in the wall of the GI tract, you really want to think about necrotizing. enterocolitis.

This kid has probably perved their abdomen, so the next step in management is immediate exploratory laparotomy, right, so that they don't become severely septic and die, okay. So this is neck, okay. Remember your pneumatosis intestinalis, preemie, okay, and you usually get broad-spectrum antibiotics, something that will cover like gram negatives and anaerobes, okay, but if they've perved the abdomen, you need to take them to surgery. And another finding you may also see is air in the wall of the gallbladder, okay, pneumobilia.

That may be another finding on imaging. Now, three-day-old newborn is brought to the ED by his mom because he started turning yellow 28 hours ago, so he's three days old. He was born without complication.

Physical exam is completely normal and notable only for scleral icterus and a yellow hue on his face, okay? Vital signs are within normal limits. Hemoglobin is 19. Okay, that's normal.

Remember, kids in general, they're sort of hypoxic in utero. So they make more EPO than normal. So they tend to be a little more on the high side with hemoglobin.

So that's totally normal. Now, T-belly is 9. D-belly is 0.7. So this kid has an indirect hyperbilirubinemia. And they say that the white count and other serum markers are within normal limits.

And COPE's testing is negative. What does this kid potentially have? So this kid has jaundice, and it's probably physiologic, because again, the hyperbilirubinemia is not a direct hyperbilirubinemia. So I'll say for your exam, you want to be able to differentiate. physiologic jaundice from pathologic jaundice, okay?

And you're very good at writing questions to sort of mess with your head, okay? So don't be one of those victims. So here's the thing.

This kid has physiologic jaundice. Physiologic jaundice is always an unconjugated hyperbilirubinemia, okay? If you see a conjugated or a direct hyperbilirubinemia on your exam, okay, that is not physiologic jaundice.

That is always pathologic, okay? And the big thing you probably want to think about is biliary atresia, okay? and lower yield diagnosis, there may be like a colidococcyst.

But in general, the first thing you want to think about is biliary atresia, okay? Now, what are the things that tell you that, okay, this is more pathologic jaundice than physiologic jaundice? If the person has jaundice in the first 24 hours of life, that is always pathologic, right?

So if it's a 19-hour-old newborn with jaundice, it's pathologic, okay? If it's a conjugated hyperbilirubinemia, it's always pathologic as well. Okay. And in general, you just get bilirubin levels, check the hemoglobin.

You can get a COMS test, right, to make sure it's not like some kind of RH or ABO incompatibility that's causing this problem. And usually you just supportive care. Usually on exams, it's like single digits indirect hyperbilirubinemia. You don't need to do anything magical.

Okay. But if it's kind of severe, if it's like 12, okay, you may want to go ahead and do phototherapy because phototherapy converts indirect. bilirubin to something that's soluble that can easily be excreted. Now, again, because this is so high yield, right, I don't want you to get this wrong in an exam.

Let's just spell it out again. Physiologic jaundice is always unconjugated, okay? It's always an indirect type of bilirubinemia.

Doesn't show up in the first day of life, okay? And it's usually, again, less than 13 pretty much on exams, okay? And it doesn't rise very fast, right?

So if it rises from like 12 to 24 the next day, that is pathologic. You want to do something about that. okay and usually results pretty early uh you can perform phototherapy and basically the problem is kids have a transient udpgt deficiency as they're young okay but as they get older they have enough enzyme activity and they conjugate all the bilirubin remember this is also why you don't want to give chloramphenicol to a neonate right because they have very low levels of udpgt and they could run into problems with that um if they also want that to be pretty nifty there now i'm thinking about it um if a kid has kreglin-ahar type 1 You also probably want to avoid chloramphenicol in those kids, because in Kreglana heart syndrome, you have a UDPGT deficiency, so you're not very good at metabolizing things that require UDPGT, like chloramphenicol, for example.

That would be a very off-putting question that I'd probably predict many people getting wrong. Now, some more things with pediatric jaundice. If you see a conjugated hyperbilirubinemia, also a direct hyperbilirubinemia. Big thing, I'll say like 99% of the time, the thing you want to think about is biliary atresia. If that is not an answer choice, think about a colidoco cyst, okay?

And if the hyperbilirubinemia is a... conjugated hyperbilirubinemia, phototherapy is not going to cut it, okay? It does not do anything. So resist the temptation to pick that kind of answer, okay?

Now, indirect hyperbilirubinemia tends to be more common, okay? It can be a physiologic jaundice kind of picture, or it could also be from Rh incompatibility, right? So those antibodies are destroying the kid's red cells, okay? So the kid is spilling a ton of hemoglobin into the serum, right?

So They'll usually have a positive COOMS test because you are detecting those bad antibodies. And your serum haptoglobin will usually be low if you have some kind of intravascular hemolysis business going on. Now, other causes of, again, an indirect hyperbilirubinemia can be a structural problem like hereditary spherocytosis. Remember, those people tend to have elevated MCHC on a CBC. And...

Classically for step one, you thought the osmotic fragility test. That may be an answer on your exam. But don't be disturbed if you see the EOSIN 5 malamide test.

It's one of those tests you can actually use to detect hereditary spherocytosis. And again, red cells that have, they don't have that central pallor. And it's like a spectrin-anchorin defect. And remember, it's autosomal dominant inheritance. And I talk about some other enzyme problems that could cause pediatric jaundice, presenting again as an indirect hyperbilirubinemia.

Now, what if you get this kind of bizarre question, right? Where they say like this kid has an elevated hematocrit, okay? And they say that the COOMS test is negative, okay?

And you see that you're like, wait, what is this, right? You're like, wait, how can a kid have a super high hematocrit and have a negative COOMS test? One thing you may want to think about there is some kind of mechanism that delivers extra blood to the fetus.

Like a twin-twin transfusion or a maternal-fetal transfusion. In general, for that, the extra blood the kid is getting is what is shooting up their hematocrit levels. Usually for that, you do something called, I'm trying to remember, I think it's called the Klyhauer-Becky test. or something like that.

I'll encourage you to look that up on your own. It's one of those weird tests that you can use to detect maternal blood in the blood of the fetus. I think it's Kleyhauer-Becky. So K-L-E-I-H-A Kleyhauer-Becky. I just hope I'm not remembering this wrong, but I think this is what you used to make that diagnosis.

Okay. Now, again, phototherapy. One thing they may do is they may say, okay, you've tried phototherapy, or the bilirubin levels are like super, super high. You may want to go ahead and perform a partial exchange transfusion, okay? Because if that indirect bilirubin deposits in the brain, okay, that can cause permanent neurologic damage.

Okay, so again, this is just a slide that sort of puts everything together. It's a quick summary of what I've already talked about, so I'll let you go over that on your own. Okay, now a newborn has a blueberry muffin rash and a right upper quadrant mass, right?

So like hepatosplenomegaly, potentially. A bilateral white reflex is observed on a fundoscopic exam. You hear a continuous machine-like murmur, excuse me, when you excitate the chest.

And they tell you that otoacoustic emission test in his neck is abnormal three days after delivery, right? So this is a test for hearing loss in kids. Now, mom had poor prenatal care. and a febrile illness at 13 weeks with a rash. The baby is small for gestational age.

So this will probably give you the answer. What do you think this is? Very good, right?

Rubella, right? So remember, rubella has an association with the PDA, patent ductus arteriosus, okay? And it may also cause bilateral cataracts, okay? Although cataracts in a kid is not always rubella, you should also think about essential galactosemia, okay?

From the galactose 1-phosphate urinal transferase deficiency. Okay, so rubella has this classic associations, right, like the cataracts, the blueberry muffin rash, although you can also find that in CMV, okay, PDA, right, so patent ductus arteriosus, they may have hearing loss, and they usually have a symmetric IUGR, okay, so this is congenital rubella. Now, a newborn with a triad of chorio-rhythmitis, right, so like eye problems, hydrocephalus, and intracranial calcifications, the mnemonic here is CHI.

So like, coronary arthritis, hydrocephalus, intracranial calcifications. And mom cleaned out the cat litter box while she was pregnant. What is this?

Toxo. Very good. Right. So this is toxoplasma gondii. OK.

In general, you want to remember your triad. OK. They almost always put this triad on exams. OK. And the thing is, they may put CMV as an answer choice to sort of lead you astray.

OK. The thing that will tell you that, oh, this is not toxo. This is CMV.

Because the thing is, CMV can also cause all these findings. OK. It's the distribution of the calcifications that guides you towards CMV on exams. If you see the buzzword.

periventricular calcifications on your exam, okay? That is 100% of the time on exams, CMV, okay? So again, you should not clean out the cat litter box if you're a mom, okay? So this is congenital toxo.

You treat with anti-folate drugs like pyrimethamine and sulfadiazine, okay? Remember, because these are anti-folate agents, they can cause bone marrow suppression, so you may want to rescue that kid with lucovorin, okay? You may also use spironicin if the mom has infection during pregnancy, okay? And again, CMV, periventricular calcifications, and you can treat that with ganciclovir, okay?

Remember, ganciclovir can cause bone marrow suppressions as well. Now, a newborn with snuffles, this is the dead giveaway on exams. Newborn with snuffles, and they have like a saddle nose deformity, they have anterior bone of the lower limbs, that's like saber shins, and they have like weird teeth where they have like an upward indentation.

What is this? This is syphilis, right? So the dead give a one exams.

He's a newborn with snuffles. Okay, that's syphilis. And those snuffles don't say, oh, it's a kid.

Let me just wipe it off with my hands. Don't do that, right? You're just auto-infecting yourself with syphilis. Those snuffles actually are teeming with a ton of spirochetes. So you want to avoid that, okay?

So this kid has congenital syphilis. And those are the findings like the saber shins, saddle nose deformity, Hutchinson's teeth, and all that stuff. It's syphilis and you want to do your RPR VDRL testing for screening. OK, but to actually confirm the diagnosis, you can do the treponemal tests.

OK, like the big words you want to remember is like MHATP, FTA, ABS and all that stuff. And if mom has this disease in pregnancy, you give up penicillin. OK, now most people know penicillin syphilis. Big deal. Right.

So they may try to get you to think a little further and say, oh, mom has. severe allergy to penicillin. And they say, what's the next best step in management?

Pregnancy with syphilis is an indication to desensitize the patient and still give the penicillin. There's no other drug that works well for syphilis in pregnancy except penicillin. Now, other high-yield congenital problems, like HSV, you're treated with acyclovir. Remember, it can cause sepsis or meningitis in the neonate.

Okay. And the classic presentation is they'll show you CSF findings and you see like a ton of red cells in the CSF. Okay. Like 500, 600 red cells in the CSF. If you see that, think about HSV.

Okay. And one thing I guess I'll give you as a bonus for your OB-GYN shelf. If a mom has herpes, okay, and you find visible lesions on the vagina, okay, that's an indication for a C-section.

But if you don't see any lesions on the vagina, you can proceed with a regular vaginal delivery. And then, again, any notable infection of a neonate, right? So as by definition, first 28 days of life, think more group B strep, right?

Strep egalactiae, okay? And then if they describe a kid that, let's say, mom ate like deli meats or soft cheeses, and this kid is like stillborn, and they tell you that, oh, there's foul-smelling amniotic fluid. and the kid has like abscesses everywhere, think about congenital listeriosis, okay? Remember listeria can cover that with ampicillin, okay?

But the buzzword there is granulomatosis infantis septicum. Now, a five-year-old is brought to the pediatrician for a routine well-child visit. Height, weight, and head circumference have tracked consistently in the 30th percentile. Blood pressure is 98 over 60, heart rate is 71, respiratory rate is 13. BMP CBC with differential is within normal limits, okay, with marginally decreased sodium.

Physical exam is totally fine, although the patient says, oh, he's itching a lot around his scalp. And you look at the scalp, okay, you see patchy hair loss, and it has no specific distribution, okay. And then you do a hair scraping with KOH, potassium hydroxide, and you find organisms with long tubular structures. What is your next best step in management, right? So you want to give an oral antifungal, right?

So what does this kid have? Tinea. Very good.

This kid has tinea capitis, okay? So, this is one thing they'll try to trip you up with on exams, right? Because many of these fungal skin infections, you can give the topical stuff and it goes away, okay?

But there are two tinea's that are not treated with topical stuff. Tinea capitis, right? So, like head fungus or tinea anguilla, nail fungus, okay?

Those things do not respond to topical therapies, okay? So, you go more with the oral stuff. Okay, oral griseofeuvin.

Another answer on exams may be like oral trabinophene. Okay, those may be answers on an exam. So, oral griseofeuvin, tinea capitis, you probably want to know the bugs that could cause the tinea, right? Like microsporum trichophyte. This is probably the more common exam answer.

Okay, and again, head tinea or toenail fungus, you treat that with oral antifungal therapy. Okay, remember, tinea vesicular is the one you treat with topical. selenium sulfide, okay? Trichotillomania, right? So if they describe a kid that has, like, some kind of psychiatric disorder, okay, and they have, like, alopecia, think about trichotillomania, okay?

One thing they could really try to, like, mess your head with on exams is to describe a kid, they say, like, oh, you perform a physical exam, you see alopecia, and this kid has, like, a two-day history of, like, nausea and severe vomiting, like, GI obstruction symptoms, think about a bezoar. Okay, that kid just swallowed hair and it has basically clogged off the GI tract. And then don't forget alopecia areata.

Okay, it may show up as a diagnosis on your test as well. Okay, now a four-year-old female presents with fever and pain with urination. This is a second UTI in the past six months.

Physical exam is notable for CVA tenderness. and a palpable left upper quadrant mass. Urine culture obtained through a suprapubic calf, or you could say I guess a midstream cling catch, grows 120,000 colonies of gram-negative lactose-fermenting rods. So what does this kid potentially have? CVA tenderness.

Very good, right? So this kid has pylon, okay? What if it's a newborn with fullness on suprapubic palpation?

What are you thinking about here? So like lower abdominal fullness in a newborn. What may be causing that obstruction that could be presenting like this? Posterior urethral valves.

Very good. Posterior urethral valves. Okay. So this kid has pyelonephritis. Okay.

And it may have arisen from a VUR, right? Like vesicle ureteral reflux. Because the thing is, kids do not...

10, especially kids of this age, like females less than five years old or like any male kid, okay, like a male young kid, they generally do not get UTIs, okay? So if you're getting UTIs or recurring UTIs, you want to think about some kind of anatomical problem that may be predisposing them to that, okay? So this kid, you want to go ahead and treat.

Pylo is pretty serious. So those kids are... deserve treatment, but once you get them over the hump of whatever infection that's raging, you want to go ahead and do a VCUG, okay, avoiding cystourethrogram, to detect, roll out any kind of anatomic cause of the recurrent urinary infections, okay, and again, for a pilot, usually on exam, ceftriaxone is a pretty safe answer, okay.

Now, posterior urethral valves, again, it's probably the most common cause of obstruction in a newborn. It can present as oligohydramnios because the kids are not peeing out amniotic fluid. And you can very quickly relieve the obstruction by placing a suprapubic cath.

But in general, your next step in moneymends is to get a VCUG, avoiding sister urethrogram, and you'll detect the lesion pretty easily on imaging. Now, if, for example, a kid has a pretty toxic presentation, they may try to say on exams, oh, like good outpatient follow up, whatever. No, don't.

OK, go ahead and admit the kid to a hospital. OK, in fact, for pilo, a kid with pilo should not be sent home. OK, they don't know outpatient follow up.

They need to be in the hospital to get IV medications. OK, another thing to watch out for. Right.

If a kid is like very dehydrated, they're like vomiting, they're like super young, like a new neat. where you maybe cannot give them oral medications, okay, go ahead and give those kids IV meds, okay, because they may say like, oh, a kid with pylo, vomiting, blah, blah, blah, and they may say like, oh, give this IV form of this medication versus the oral form. You want to go ahead and give the IV form, okay, because they are vomiting, they can't keep the medication down.

So those are very good indications on exams to go with IV meds, okay? Does that make sense? Awesome. Okay, now CP, cerebral palsy, it's a motor disease, okay, it doesn't progress, okay, no one really knows what causes it. In general, the most common kind of cerebral palsy, there are many types, is the spastic kind, okay, and for the spasticity, you can just give them muscle relaxants, right, like baclofen, it's a GABA-B receptor agonist, or you can inject Botox.

Okay, so next question, six-month-old male comes for his routine peds visit, His weight at birth was 5 pounds. His current weight is 9 pounds. His first bowel movement was at 47 hours after birth.

That's not very good. He has experienced 3 bouts of pneumonia, although 48 hours is the cutoff for failure to pass meconium. But, he has experienced 3 bouts of pneumonia since birth and has foul-smelling greasy stools. He was recently treated for rectal prolapse. That's a strong association.

And his mom has noticed that he always tastes salty when the heater is turned on. or when he's rocked in the sun. Physical exam is notable for digital clubbing and recurrent nasal polyps, which have not been well controlled with intranasal steroids.

What does this kid have? Cystic fibrosis. Very good. Okay, so again, foul-smelling greasy stools because they have pancreatic insufficiency. Okay, so they are not reabsorbing fat.

Okay, so that's why they have like the diarrhea with the nasty smelling stools. Okay. Again, they have a thick airway secretion, so they get recurring pneumonia. Again, before the age of 20, think more staph aureus.

After the age of 20, think more pseudomonas aeruginosa. Okay. And again, nasal polyps in a kid, you really want to think about CF on exams.

And remember your sweat chloride testing. They may also say, oh, which of the following serum findings is most likely in this disorder? Okay.

They will have low levels of trypsinogen. Because remember, trypsinogen is a pancreatic enzyme. So if your pancreatic ducts are clogged off, you're not releasing trypsinogen into the bloodstream.

So those kids will have low levels of trypsinogen. But sweat chloride testing is usually pretty diagnostic. Now, CF.

Again, CFTR gene mutation. The most common one is the Delta F508 mutation. It's a chromosome 7 problem, autosomal recessive. And in general, the way you treat... is you do like pulmonary therapy right you can give them like N-acetylcysteine to sort of break up remember this one breaks disulfide bonds this one breaks phosphodiester bonds to break up the secretions you can try to cover them in greens gram negatives like Pseudomonas with an amino glycoside okay alternatively you may also want to Give them back what they are missing, right?

Like pancreatic enzymes, you want to give them these fat soluble vitamins, okay? And you also want to do like chest physiotherapy, okay? Now, greasy stools in a kid and they say like, oh, this kid has like rickets, for example, from a vitamin D deficiency. And they don't describe having respiratory problems.

You also probably want to think about celiac disease with that, okay? Celiac disease is actually a pretty common condition in kids. So don't think of, because we've all been trained to think, oh, celiac disease, older person, that is not always true in the real world, okay?

So, you may actually get a celiac disease question on exams, okay? And remember your anti-tissue transglutaminase. In those cases, they just stop taking gluten and they should be fine.

And don't forget the association with rash, vesicular rash on the extensor surfaces, your dermatitis herpetiformis. For that, you can actually give Dapsone, okay? Dapsone is pretty good treatment for dermatitis.

herpetiformis. Although the other thing I will say with that is kids with celiac disease, they have an increased risk of lymphomas of the mucosa of the GI tract, okay? It's called like EATL, enteric association of the T-cell lymphoma, okay? Now, let's do some quick questions here. So, given the following clinical scenarios, what's the most likely diagnosis?

Okay, I'm going to speed up here. So... A 17-year-old female is brought to the pediatrician by her mom with complaints of soft throat, high fevers, abdominal pain, and vomiting that began suddenly yesterday. Physical exam is notable for tender anterior cervical lymphadenopathy, redness of the hard and soft palate, and a sandpaper rash along her upper extremities.

What is that? Very good, right? That's strep pharyngitis, right?

So sandpaper rash, anterior cervical lymphadenopathy. But notice, in this question, I do not put conjunctivitis, okay? If I put conjunctivitis, you're probably thinking more about Kawasaki's disease.

Now, three-year-old male is brought to the pediatrician by his mom with complaints of sore throat that started over three days ago. He has had a persistent cough and rhinorrhea for the past week. What is this? Very good, right? This is just a viral URI, right?

So usually mild fever, persistent cough, runny nose. Okay, now five-year-old female is brought to the pediatrician by her mom with complaints of soft throat and watery eyes that started over three days ago. Okay, now, so this is a viral-like picture, but this kid has severe conjunctival erythema. Okay, and let's assume I tell you that this kid also has abdominal pain.

What is this? This is adenovirus. Very good, right?

So adenovirus, classically, in fact, they call it pharyngoconjunctivitis, right? So a pharyngeal infection, right? Soft throat with red eye, and it can also cause abdominal pain. And one of the unusual thing that adenovirus may present with on an exam is hemorrhagic cystitis. It's actually one of those bugs that can cause hemorrhagic cystitis, I guess, in addition to schistosoma hematobium.

Now, A six-year-old male is brought to a pediatrician by his mom with complaints of sore throat and high fevers. Physical exam is notable for erythematous lesions with central clearing on the palms, right? So a rash on the palms.

And you'll see ulcerative lesions in the mouth. What is this? Koksaki. Very good, right?

So this is hand, foot, mouth disease. Remember the causes of a rash on the palms and soles, right? So like there's this step one mnemonic, you drive Kawasaki cars. with your hands and feet, right?

So like Coxsackie A, Rickettsia Rickettsia, Rocky Mountain spotted fever, and secondary syphilis, okay? And then Kawasaki's disease. So this kid has hand, foot, mouth disease caused by Coxsackie A. And the lesions they have on the mouth, that's a hepangina, okay? Now, a 14-year-old female is brought to a pediatrician by her mom with complaints of sore throat and mouth fevers for the past two weeks.

She has been extremely tired and unable to participate in competitive lacrosse over the last three weeks. Physical exam, so notes the age range. Youngish person that may be sexually active. Now, physical exam is notable for generalized cervical lymphadenopathy.

There is fullness on palpation of the left upper quadrant, so think of the organ that's there. Now, initial treatment with amoxicillin triggered a severe dermatologic skin reaction, which led to the drug being discontinued. What is this?

Very good. This is Mano. Okay, EBV.

Good. Now, a two-year-old male is brought to a pediatrician by his enlightened parents with complaints of a sore throat. They live in Palo Alto, California. So let's assume they are very enlightened.

Now, a HENT exam, this would also be an immigrant on exam. So a HENT exam is notable for the presence of a grayish pseudomembrane in the posterior pharynx with what looks like a bull's neck. What is this?

diphtheria very good caused by corinibacterium diphtheria right so let's assume this kid did not get the vaccine okay so diphtheria infection okay so those are your answers strep pharyngitis again it usually does not show up in kids less than two years old on exams usually like in a six or seven year old okay and again look at your age ranges it usually helps on exams and your treatment for strep pharyngitis is you can give far penicillin, like ampicillin, amoxicillin, penicillin, whatever. If the kid is pen allergic, you can give a macrolate, okay? That should also help. And in general, by giving antibiotics, you decrease the risk of rheumatic fever, okay?

But antibiotics do not decrease the risk of post-infectious glomerulonephritis. And then I put the center criteria there. You can go through that on your own.

And then viral pharyngitis, adenovirus. Again, if you see conjunctivitis with pharyngitis, don't think strep pharyngitis. Think more of adenovirus. That's why it's called pharyngoconjunctivitis. And then we've talked about hand-foot-mouth disease.

Remember, Coxsackie B is a pretty common cause of myocarditis. in human beings in general. So it's probably the most common cause of viral myocarditis.

And remember, for EBV, you perform the monospot test. Remember, it's associated with many malignancies like nasopharyngeal carcinoma. It can also cause a Burkitt's lymphoma, where you see the starry sky pattern on microscopy. Remember, you're associated with an 814 translocation for that. And if you see a monospot negative mononucleosis syndrome, I really hope you're thinking about CMV with that.

Okay, so the theory, and let me just say a few words on abscesses, right? So there are two, like... airway abscesses you want to think about. You want to think about either a peritonsillar abscess or a retropharyngeal abscess on exams, okay?

Now, the thing is, in general, if they want you to think more of a retropharyngeal abscess, they will give you an image on the test, okay? Because they're sort of hard to differentiate on exams, okay? Although, in general, a retropharyngeal abscess is usually like pretty toxic.

It's usually like a pretty toxic presentation. Okay, it's sort of similar to epiglottitis. But peritonsillar abscess, they will describe a kid that's like relatively normal, has trouble swallowing, like troubles with eating.

And they may tell you that, oh, on physical exam, there is a deviation of the uvula. If you see that buzzword, you should probably think more peritonsillar abscess than retropharyngeal abscess. Okay, retropharyngeal abscess, they usually show you like a lateral neck x-ray. The pre-vertebral tissue, it's something you should probably look up and like look at an image online, but the pre-vertebral tissue is usually pretty thick, like there's a size criteria that people that are in radiology use pretty commonly for that.

Now, anaphylactic reactions to a vaccine, right, so there are just some weird things they put with vaccines on exams, right, so if a kid has like an immunodeficiency disorder like SCID, for example, or they have the George syndrome, where they have a T cell problem, you don't want to give those kids live attenuated vaccines, okay? And remember your live attenuated vaccines, right? Like MMR, the injectable influenza vaccine is fine, okay? But the intranasal one is live attenuated, okay? And again, if a kid is less than a year old, you also should not give them a live attenuated vaccine, okay?

And if a kid has very high fevers, like a very severe illness with very high fevers, you should also not give them a vaccine then. OK, you can give them when it resolves. Or if a kid has like a very severe allergic reaction to a vaccine, that's also a contraindication to that to that vaccine.

And then if a kid has no spleen, right, you want to vaccinate them against the shin organisms. Right. So shin.

That's the nice mnemonic there for strep pneumo, right, H flu, okay, and mycelium, okay? And a bizarre thing they may actually do on exams is to say, let's assume they describe a kid that has a history of paroxysmal nocturnal hemoglobinuria, where they have problems with GPI anchors, right? So their red cells constantly explode.

And the treatment for that disorder is eculizumab. right, eculizumab is a monoclonal antibody against C5, okay? The thing is, if you remember back from step one immunology, kids that have a C5 deficiency, they tend to get recurrent infections with Neisseria, okay? So one weird question they may give you on exams is they may say, a kid has a history of PNH, they're on eculizumab therapy, and then they say, what's the next best step in management? And it may be to give them a Neisseria meningitis vaccination, okay?

Because eculizumab induces a pharmacologic C5 deficiency. So that kid can have recurrent mycelial infections. And then I've talked about the rotavirus vaccine. And don't give it to people that have had intussusception or Meckel's diverticulum. And then I talk about some feeding associations, like if a kid is exclusively breastfed, you should supplement vitamin D.

Milk contains IgA, okay? But if a mom has like herpes or a mom has like HIV or a mom has or the kid has essential galactosemia, right? So a galactose 1 phosphate urinal transferase deficiency, those kids should not breastfeed, okay?

So that they don't have very severe symptoms, okay? And one unusual thing. No one really knows why this is, but it's just one of those things that show up on exams pretty commonly.

One of the common causes of death in kids with galactosemia is actually sepsis with E. coli. E.

coli sepsis, that's a big association you want to know. And then I talk about milk carriers, you can read that on your own. And again, don't forget ulcerative colitis and Crohn's. You may say, hmm, divine, those are old people diseases. No, they're not.

Okay. They may actually make that a pediatric disease on an exam. Okay. So the NBMA has been known to give you all people diseases in the pediatric population. Okay.

So just be mindful of that. Now, SIDS, you want to know the risk factors, right? So if you live in a home where the parents smoke or the kid sleeps on a soft surface, you want your kids to sleep on hard surfaces.

Or if the kid is sleeping prone, that's not good. Remember the back to sleep campaign. Okay.

Those are all risk factors for For SIDS, Infant Distress Syndrome with Infants. It's probably one of the more common causes of death in kids less than a year. Okay, now, child with nasty coughing episodes. Okay, the child... vomits, right?

So these are big buzzwords here. The child vomits after severe coughing fits, okay? And the physical exam is notable for subconjunctival hemorrhage. You treat with a macrolate like erythromycin, okay? And this is a bacterial infection.

And if you think about most bacterial infections, right, when you get a bacterial infection, you have like a severe elevation in your white count, but it's usually with neutrophils because it's bacteria. But this one is a bacterial infection where you get... lymphocytic pleocytosis.

And you can grow this on like Bordeigian or Oregon low media. What is this? Very good, right?

So this is protosis. So this is whooping cough. So if they describe a kid that like cough, cough, cough, that's whooping cough. That's the classic presentation.

And usually the kids will have like runny nose, mild fevers and all that stuff. And then they'll have the cataract phase where they have like the cough. And the cough is usually so bad and it's so long lasting. that these kids, they may vomit.

So if you see a question about a kid that either vomits after a cough or the kid has such severe coughing that they have hemorrhage in the eyes, you really want to think about bordetella pertussis, okay? And for close contacts, you want to give them macrolates. Macrolates are also the treatment for the disease, okay?

And it's spread via respiratory droplets. Okay, now two-year-old presents with bilateral swelling above the jaw. Okay. His symptoms started two days ago as neck pain, fever, and a severe headache. Okay.

And a sequel of this infection is inflammation of the testes that could cause infertility, right? So big, so like a kid's like this, and they have orchitis. What may cause that?

That's vaccine preventable. Mumps. Very good, right? So mumps.

Okay. Mumps. And mumps is one of those weird causes of pancreatitis.

Now, present in 2% of the population, 2 feet from the ileocecal valve and contains 2 types of tissue. Okay, you probably know the answer already. Painless rectal bleeding.

You can make the diagnosis with a technetium scan. Okay, and this can actually serve as a lead point for intrusoception. So, a patient that has this disorder, you may not want to give them the rotavirus vaccine.

What is this? Very good. This is Meckel's diverticulum. Okay, it's a true diverticulum. Okay, in general, you'll make the diagnosis with a Meckle scan, although they may not put Meckle scan on the exam because they'll give away the answer.

Okay, they may put a technetium 99M scan. Okay, and in general, you'll go ahead and surgically resect the lesion and it goes away. And then don't forget that a Meckle diverticulum can serve as a lead point for intussusception.

Okay, now a two-year-old is angry with mom for not giving him breast milk. breast milk okay his mom notices the child suddenly passing out on the carpet he regains consciousness within 15 seconds this is a pretty bizarre question do you know what it is so this kid may be holding his breath exactly okay so this is a breath holding spell um you probably don't need to worry about the types of breath holding spells but i just put that here for you uh It has an association with iron deficiency, so replenishing iron may be a potential answer choice. But it's pretty benign.

The kids are not going to die from this. Their medulla will sort of take over and they'll start breathing spontaneously. Usually, it's either the kid is super angry or they're super scared or they see blood and then they pass out.

It's like a breath-holding spell. Now, a female with short stature. Webbing of the neck, so like congenital lymphedema, if you may, and infertility.

This kid has coarctation of the ureter and horseshoe kidney, and this is a common cause of primary amenorrhea. Very good, right? So this patient has Turner syndrome.

Okay, remember it's 45XO. Okay, if a female has like mosaicism, they may have less severe presentation. But don't forget your classic associations, right?

So like short stature, they may have like webbing of the neck. They may be infertile because they have like strict gonads, okay? These are the common cardiac associations. They may also have bicuspid aortic valve. I didn't include that here.

Horseshoe kidney is also pretty common, right? inferior pores of the kidney fuse and the stock under the inferior mesentery artery okay in general for to make them grow like to grow you can give them growth hormone okay and then for them to have a secondary sexual characteristics you can give them our female hormones now super small testicles okay so small testicles six-foot tall male with gynecomastia and infertility and the IQ is normal ish but it's low vision is otherwise normal what is this Right, so this is Klinefelter, okay? This is another genetic problem, right?

So 47XXY, okay? So it's usually a tall male with gynecomastia and infertility, okay? Think about Klinefelter.

And then if they describe a tall male that has a lot of acne and is super violent, you want to think about 47XYY, okay? This one is super low yield, so I'm just going to leave that there. You can go over it on your own, the triple X karyotype.

It has like a very weird specific association, but... We don't have time for that. Okay, so let's do some triggers. So we're getting to the end of this. We'll probably stop at some point pretty soon, actually.

Okay, so let's do some quick triggers, right? So these are genetic, toxin, metabolic-ish disorders. So this one's kind of hard, okay?

This is why you see me tell people that it's actually good to do PEDS right after step one because all these biochem-like presentations actually show up pretty liberally. on tests. So, intellectual disability, low birth weight, microcephaly, and a high-pitched cat-like cry. So, this is Cree-Dushat syndrome, okay? Cree-Dushat syndrome, okay?

And usually for these diseases, you don't need to know everything about them. You just need to know like one high yield important thing, okay? Now, obesity, right?

So, a very fat kid, usually it's male, okay? Infantile hypotonia and a chromosome 15 deletion. Very good.

This is Prader-Willi, okay? It's a chromosome 15 problem, and there are two genetic principles you want to remember, okay? We don't have the time for me to explain, but think of, or you can listen to a podcast I made where I explained it, but basically, think of this in relation to uniparental disomy. It's one of those weird things that may show up on a test.

Uniparental disomy. And it also has an association with genetic imprinting. And it's chromosome 15. Okay.

Those are the big associations you want to know for Prader-Willi syndrome. Now, this is the analogous problem in females. So, happy mood, inappropriate laughter, ataxia and chromosome 15 deletion. Angel man, right?

So, angel man. Angel man like mom. Prader-Willi like pop.

So, this isn't a guy. Now, elfin faces. If you see that buzzword, what are you thinking about?

William syndrome. Awesome. Okay. Now, CGG repeat. And this kid has a long face and large ears and very big testicles.

Fragile. Very good. Fragile X syndrome.

Okay. We've talked about this one. Smooth philtrum. What does that make you think about? Fetal alcohol syndrome.

Remember the association of the VSD and ADHD. Okay. Now, the tricuspid valve is displaced downward and you have hypoplasia of the right ventricle. So, apical displacement of the tricuspid valve.

And let's assume your mom is being treated for bipolar disorder. What kind of toxicity is this? lithium.

Okay. Now lateral neck mass that does not move with swallowing, and it comes from ectoderm. It's what kind of cyst?

Okay. So it's a brachial cleft cyst. Okay.

You've been doing very good so far. Now, yeah, this slide is probably the more challenging slide. This is probably pretty good flush card material.

Now, midline neck mass that moves with swallowing, and it may have ectopic thyroid tissue. Very good. This is a thyroglossoduct cyst.

Awesome. Now, hypopigmented macules, chagrin patches, adenoma sebaceum, infantile spasms. Remember that thing I said you treat with ACTH or vigabatrin. Cardiac rhabdomyo, I'll say this is probably more cardiac rhabdomyoma, so let me correct that. A cardiac rhabdomyoma and a renal angiomyolipoma.

What's the syndrome here? tuberous sclerosis, and remember, autosomal dominant inheritance. Now, severe fasting hypoglycemia.

So this is basically the glycogen storage diseases. I know these are usually kind of tough, but bear with me. So severe fasting hypoglycemia, lactic acidosis, hepatomegaly, gout, and this kid may have kidney symptoms, but they have no muscle problems.

What do you think this is? So this is von Gierks, okay, glycogen storage disease type 1. I go over the glycogen storage diseases in another podcast, so I'll encourage you to listen to that to understand the pathophysiology, because if you understand it, you probably don't need to memorize as much, okay? But this one is pretty classic.

What's the glycogen storage disease that presents with heart failure in kids? That's Pompeii. It's very good.

That's GSD type 2. Okay. Now, mild hypoglycemia, hepatomegaly. So this kid has liver symptoms and muscle symptoms, but they have no lactic acidosis. This is Corrie disease. Okay.

So it's a debranching enzyme defect. And I'll give you like a very quick trick for remembering this at the end. Now, no lactic acidosis, but they have muscle symptoms predominantly. Mercardals, right?

So use the M in muscle for M in mercardals. That's glycogen story disease type five, okay? So here's the quick algorithm you use for this. If the kid has muscle and liver symptoms on the exam, okay, that is story disease. That's GSD type three.

So you have muscle and liver symptoms for the type three GSD, okay? The one that is associated with only muscle symptoms is type five, mercardals. Okay.

Type 1 of Ongeric's disease has only liver symptoms. Okay. And then the one that has heart failure is type 2. So those are the big things you want to look out for.

Liver only is type 1. Muscle only is type 5. Heart failure is type 2. Muscle and liver is type 3. Okay. Cori's disease. So those are your answers. Now, on December 1st, a four-year-old immigrant has a temperature of 102, cough, bilateral ocular erythema, and a runny nose.

Okay. Now, physical exam in the office reveals whitish spots on the buccal mucosa. I'll tell you those are carpalic spots.

Okay. And there's a red maculopapular rash on the face that spreads down to the trunk. Okay.

And after some supportive care, the rash resolves. What is this? Very good. This is measles. Okay, this is rubella.

Okay, do not confuse measles. Measles is rubella. Don't confuse it with German measles.

I know it's kind of frustrating. German measles is rubella. German measles, rubella. Measles, regular measles is rubella.

Okay, so don't forget your three C's, right, like your cough, choriza. Choriza is just a very expensive term for a runny nose. Okay, and conjunctivitis. And the classic presentation is they have like rhinorrhea, like upper respiratory, like...

symptoms like mild fever, and then they have your other Cs, and then it resolves, okay? And there's really no treatment for this, but the thing that may be helpful is vitamin A, okay? Supplementing vitamin A may actually help here.

And if they describe a kid that had like, maybe like a past measles infection, and now they're like not doing well in school, or they have like this weird neurological defects, you really want to think about subacute sclerosin panencephalitis. It's essentially like 100% fetal. Most kids do not recover from that.

Now, slightly similar presentation, right? Where the rest starts on the head moves downward. But now I put this buzzword, posterior auricular and suboccipital lymphadenopathy. What do you think this is?

Very good. This is rubella, okay? Rubella, right? So German measles, right? So remember that rubella has the association with the patent doctor's arteriosus, right?

So like the continuous machine-like murmur, okay? And roseola, right? So herpes 6. The classic presentation is they'll say like, oh, a kid had a very high fever.

It can be like up to 106 on an exam. And then you see the fever resolves. And then like three days later, boom, they have like this sudden rash, usually on the face.

You really want to think about a roseola, okay, exanthem, subethem, or like sixth disease, okay? It's a pretty common exam question. Now, this one, I just put this here.

Every, usually most questions on the shelf exams are like pretty classic presentations, but you may put some things that you've probably never found in any resource. So I'm just going to leave this here for you to just sort of keep in mind. If you remember what the disease means, you also remember...

all the findings, right? So it's called FAPA syndrome. It's a periodic fever syndrome. So these kids, they have fevers. The fevers come on a schedule.

They come like every like three to five weeks, okay? And then they'll have like ulcers in the mouth. And then they usually have like sore throat and like very severe inflammation of lymph nodes in the cervical region.

It's usually self-limited, like resolved spontaneously. But on your exam, This responds pretty well to NSAIDs. And another thing that it responds pretty well to are the IL-1 inhibitors.

Because interleukin-1 has been shown to play a very big role in the pathophysiology of this disease. So you can give like a monoclonal, like a canakinumab. It does pretty well for the periodic fever syndromes, like FAPA syndrome.

Okay. Now, a medical student rotating in the nursery notices severe scalp swelling in the newborn male. The edema from the lesion crosses. Crosses suture lines.

This is pretty classic. What is this? Very good. This is caput. Soxedanium.

Okay. Now, what if you notice scalp swelling, but it does not cross suture lines? Very good, right?

So that's the big buzzword. Crosses suture lines, caput succidanium, does not cross suture lines, that's a cephalohematoma, okay? Now, newborn female, you see like streaks of blood from the vagina. Should you be worried here? No, exactly, right?

So, mom is just, as the baby has been delivered, right, it's like you're getting, like, a withdrawal of hormones, right? So, the kid is just having, like, a newborn menstruation. It's benign, goes away, that's the end of that.

Okay, so those are your answers. That's your Abgar scoring system. It's not predictive of outcomes, okay, but you use it to guide your newborn resuscitative procedures.

And then, now we're going to pediatric cardiology. I think those are the last things we're actually going to worry about here today. But the big one is tetralogy of Fallot, right?

So you want to know your four findings. And I just put this diagram here to sort of show you the pathophysiology, right? So this is normal.

Your urinal pulmonary septum usually comes and divides the ventricles pretty nicely, okay? But if it doesn't, if you have like an equal division of your ventricles, right? You basically have, you have like a B and E.

The aortic pulmonary septum is not leading the interventricular septum. Unlike the VSE, because the outflow tract on the right side of the heart is smaller, you have pulmonic stenosis. The order is literally overriding these two ventricles.

Okay, because this pulmonic outflow tract is stimulus, we have hypertrophy of the right ventricle. Okay, so remember the classic association with TED spells on exams. And you use like the bleedlocked house sections to fix this.

Okay, now, 15-year-old male presents to his pediatrician with a three-month history of fevers and pain just below the femoral head. Okay, now, this notes the age. The pain was initially well-controlled with acetaminophen, but has progressively gotten worse and unbearable over the past four weeks.

If you do labs on this kid, you find an increased ESR and a proliferation of white cells, and you look at the femur. on imaging, and you find a moth-eating appearance, and you find a concentric whirling pattern. So let me call that onion skinning, okay?

And you perform a biopsy of the lesion, and you see small blue cells. What is this? Very good. This is Ewing sarcoma.

Awesome. Ewing sarcoma, okay? Remember the association with 11-22 translocation, and you could treat this with dactinomycin.

Dactinomycin treats like Wilms tumors, Ewing sarcoma, like childhood cancers. Now, I'll give you a descriptor. You tell me what you think, right?

So these are Pete's cardio questions, right? So a newborn male, a newborn presents with cyanosis 30 minutes after delivery. And let me give you another trick here. If they describe a kid that sweats with feeding, the very first thing you should think about on your exam is a cardiac problem.

It's just one of those bizarre things you never think about, but they may introduce it in the question step. So you want to use that to focus your thinking. If a kid is breastfeeding and they say, oh, the kid is sweaty with feeds, you really want to think about a pediatric cardiac issue.

So, newborn presents with cyanosis 30 minutes after delivery. Pulmonary vascular markings are normal. An emergent chest x-ray reveals a heart that is shaped like an egg on a string. Very good.

Transposition, right? So, you want to remember this buzzword. Egg on a string appearance to the heart. That is transposition of the great vessels.

Okay. Now, a Med-19 student rotating in the nursery calls the Peds attending to report a pertinent finding. Auscultation of a three-day-old newborn with pretty good Abger scores at one in five minutes reveals a systolic ejection murmur at the left upper sternal border and a wide fixed split of the S2 heart cell. That's the buzzword.

Chest x-ray shows right heart enlargement and increased pulmonary vascular markings. Very good. That's an AST, okay? That's an atrial septal defect, okay?

Now, if you notice, I keep mentioning pulmonary vascular markings in these questions. There are just certain things I think will help you on your test, right? So if they tell you that pulmonary vascular markings are decreased, you want to think of some congenital problem that has stenosis of the pulmonic valve as part of the findings.

Like Tetralogy of Thillot will have decreased pulmonary vascular markings, okay? tricuspid atresia will have decreased pulmonary vascular markings, okay? So that's just one thing that may help you on tests.

Now remember ASDs, if you keep sending more, a lot of blood to the right side of the heart, right? That's why your pulmonary vascular markings are increased on imaging. And the most common kind of ASD is the secondum ASD, okay?

But not in Down syndrome. The most common kind of ASD in Down syndrome is the osteoprimum ASD, okay? Because those kids tend to have endocardial cushion defects, and the endocardial cushion forms like the inferior part, if you're in the anatomical position, the inferior parts of the osteoprimum. So they tend to have osteoprimum ASDs.

Now, a 16-year-old female is brought to a pediatrician with a four-year history of severe recurrent headaches. She's four feet tall and has a low posterior hairline, in addition to chronic lymphedema. on both sides of her neck. She had surgery five years ago for some kidney problem. Let's assume it's horseshoe kidney.

And physical exam is notable for a loud murmur around the left scapula. Her lower extremity, posterior tibial, and dorsalis pedis pulses are barely palpable. And a chest x-ray shows what appears to be rib indentations and a three sign just after the aortic arch. What is this?

Very good. This is aortic quartation, right? So what does this kid have? Turner syndrome. Very good.

Okay, and I'll just say this. If a kid is born and they have like immediate cyanosis, usually on your exam they'll say what is the next step in management. Just give them a PGE1 analog.

The big one that they may put on exams is alprostadil. Alprostadil is a PGE1 analog. You can also use a PGE2 analog like dinoprostone. Those can keep the doctor's arteriosus open before you proceed to surgery. Now a newborn presents to a pediatrician for his one-week postnatal appointment.

Cardiac auscultation is notable for a holosystolic murmur, best heard at the... left lower sternal border. Okay.

A loud pulmonic second heart sound and an apical diastolic rumble. This is probably the big word you're looking for. Holosystolic murmur.

And a chest x-ray is consistent with increased pulmonary vascular markings. So more blood is going through the pulmonic valve. What is this? That's a VSD.

Okay. Remember, VSDs have a strong association with fetal alcohol syndrome. So harsh holosystolic murmur. And in general, the smaller a VSD, the louder it actually sounds.

Now, a newborn with multiple facial defects and hypocalcemic seizures is transported to the Med-19 NICU for persistent cyanosis. Okay. Cardiac auscultation is notable for a single S2 heart sound.

So, let me give you the buzzwords here. Facial problems, hypocalcemia. So, let's assume the parathyroidism did not develop.

And let's say this kid has recurring viral and fungal infections. What are you thinking about here? Very good, DeGeorge syndrome.

And what is the terrible T that has a very strong association with DeGeorge syndrome? Truncus arteriosus. Very good, truncus arteriosus. Awesome.

Okay, now three-hour-old newborn is seen by a med student in the NICU for persistent cyanosis. Cardiac auscultation is consistent with a wide fixed split of the second heart cell, so that's one problem, excuse me, a harsh holosystolic murmur at the left lower sternal border. and a diminished S1.

A chest x-ray is consistent with decreased pulmonary vascular markings, and an EKG is notable for left axis deviation. This one is kind of difficult. So this is tricuspid atresia, okay? We don't have the time because I probably want to end here, but the only cyanotic congenital heart disease that ever has left axis deviation is tricuspid atresia.

Now, the pathophysiology here is the tricuspid valve basically does not develop. So there's no communication between the right atrium and the right ventricle. Okay? So for these kids to survive, they need some things to happen.

They need an ASD. so that blood can go from the right atrium to the left atrium. And then they need a VSD for blood to flow from the left ventricle to the right ventricle and then through the pulmonary system. If they don't have a VSD, they will need a PDA so that blood can go from the left ventricle, and then go to supply the rest of the body. So they either need an ASD plus a VSD or an ASD plus a PDA.

Usually the kids that have an ASD plus a PDA are the ones that will have decreased pulmonary vascular markings. But because the left ventricle is pumping to the systemic circulation and the pulmonary circulation, it gets bigger. So on EKG, you'll see a left axis deviation.

Remember, left ventricular hypertrophy is one of the common causes of left axis deviation, right? So left axis deviation, cyanotic congenital heart condition, that is tricuspid atresia, okay? So I'm going to stop here.

I will let you go through the very few remaining slides. I just put some of these metabolic diseases. I'll say they are probably very high yield for you to know, like your maple syrup, urine disease, heart knob, only thing, transcabamolase deficiency and all that stuff.

I'll say you probably get some questions right from going through these last slides. Okay, so just go through them. There's not much and I wish you all the best on your exam.

Okay, thank you.